Download as pdf or txt
Download as pdf or txt
You are on page 1of 150

Question 1 of 37

1 A 69-year-old man with a history of hypertension, diabetes, and HIV presents with a rapidly
enlarging lesion over his left shoulder. He reports a significant history of sun exposure. He has a 2
cm non-tender, raised, lightly pigmented lesion over the left shoulder on examination. Left axillary
lymphadenopathy is palpated. Biopsy of the skin lesion shows a dermal mass extending into the
subcutis, composed of strands of uniform, round, blue cells, positive for CK20 perinuclear staining,
CD56, chromogranin, and synaptophysin, and negative for S100, CK7, and thyroid-transcription
factor-1. Fluorodeoxyglucose (FDG)-PET/CT shows hypermetabolic enlarged left axillary nodes and
numerous pulmonary and liver lesions. A percutaneous biopsy of a liver lesion shows similar
findings to the skin lesion biopsy.

Which of the following is the recommended first-line treatment?

 Carboplatin, etoposide, and atezolizumab

 Avelumab

 Ipilimumab

 Ipilimumab and nivolumab

Submit
Submit
Question 1 of 37

1 A 69-year-old man with a history of hypertension, diabetes, and HIV presents with a rapidly
enlarging lesion over his left shoulder. He reports a significant history of sun exposure. He has a 2
cm non-tender, raised, lightly pigmented lesion over the left shoulder on examination. Left axillary
lymphadenopathy is palpated. Biopsy of the skin lesion shows a dermal mass extending into the
subcutis, composed of strands of uniform, round, blue cells, positive for CK20 perinuclear staining,
CD56, chromogranin, and synaptophysin, and negative for S100, CK7, and thyroid-transcription
factor-1. Fluorodeoxyglucose (FDG)-PET/CT shows hypermetabolic enlarged left axillary nodes and
numerous pulmonary and liver lesions. A percutaneous biopsy of a liver lesion shows similar
findings to the skin lesion biopsy.

Which of the following is the recommended first-line treatment?

 Carboplatin, etoposide, and atezolizumab

 Avelumab

 Ipilimumab

 Ipilimumab and nivolumab

Rationale:
This is a patient with metastatic Merkel cell carcinoma (MCC). He has several risk factors, including
older age (>50), immunosuppression (HIV), and a history of significant sun exposure. The
described pathologic findings are typical for this tumor type, including features of both epithelial and
neuroendocrine cells, particularly CK20 positivity. Negativity for TTF-1 and S100 help exclude small
cell carcinoma of the lung and melanoma, respectively. Avelumab is a monoclonal anti-PDL1
antibody with demonstrated activity in advanced MCC and is currently approved in the first-line
setting. Ipilimumab and the combination of ipilimumab and nivolumab are not approved in the first-
line setting for advanced MCC at this time. Carboplatin, etoposide, and atezolizumab are currently
approved first-line regimens for the lung's extensive-stage small cell carcinoma. It is not approved
at this time in the first-line setting for advanced MCC.

References:
Kaufman HL, Russell J, Hamid O, et al. Avelumab in patients with chemotherapy-refractory
metastatic Merkel cell carcinoma: a multicentre, single-group, open-label, phase 2 trial. Lancet
Oncol. 2016;17(10):1374-1385. https://pubmed.ncbi.nlm.nih.gov/27592805/
Next
Question 2 of 37

2 A 69-year-old man with oculocutaneous albinism (OCA) and a history of numerous cutaneous
squamous cell carcinomas presents with an enlarging right neck mass. He has a new skin lesion
over the right lower face and a large nodal mass on the right side of his neck—biopsy of the skin
lesion and fine-needle aspiration of the right cervical nodal mass show squamous cell carcinoma.
No distant metastases are identified. The skin lesion is resected, and he receives radiation therapy
to the right neck concurrently with chemotherapy. After an initial partial response, he has
progressive disease in the right neck. Fluorodeoxyglucose (FDG)-PET/CT shows increased
hypermetabolic right cervical adenopathy and numerous new pulmonary lesions. CT-guided biopsy
of a pulmonary lesion shows squamous cell carcinoma. 

Which of the following would be the best management at this time?

 Vismodegib

 Carboplatin and fluorouracil

 Gefitinib

 Cemiplimab

Submit
Submit
Question 2 of 37

2 A 69-year-old man with oculocutaneous albinism (OCA) and a history of numerous cutaneous
squamous cell carcinomas presents with an enlarging right neck mass. He has a new skin lesion
over the right lower face and a large nodal mass on the right side of his neck—biopsy of the skin
lesion and fine-needle aspiration of the right cervical nodal mass show squamous cell carcinoma.
No distant metastases are identified. The skin lesion is resected, and he receives radiation therapy
to the right neck concurrently with chemotherapy. After an initial partial response, he has
progressive disease in the right neck. Fluorodeoxyglucose (FDG)-PET/CT shows increased
hypermetabolic right cervical adenopathy and numerous new pulmonary lesions. CT-guided biopsy
of a pulmonary lesion shows squamous cell carcinoma. 

Which of the following would be the best management at this time?

 Vismodegib

 Carboplatin and fluorouracil

 Gefitinib

 Cemiplimab

Rationale:
This is a patient with now metastatic cutaneous squamous cell carcinoma (SCC). Cemiplimab is an
anti-PD1 antibody that is effective in this setting. Vismodegib is an SMO inhibitor that blocks the
hedgehog pathway's activation and has demonstrated efficacy in basal cell carcinoma. Systemic
chemotherapy has limited efficacy in advanced cutaneous SCC. EGFR inhibitors such as gefitinib
and erlotinib have also been shown to have limited efficacy in advanced cutaneous SCC.

References:
Migden MR, Rischin D, Schmults CD, et al. PD-1 Blockade with Cemiplimab in Advanced
Cutaneous Squamous-Cell Carcinoma. N Engl J Med. 2018;379(4):341-351.
https://pubmed.ncbi.nlm.nih.gov/29863979/

Next
Question 3 of 37

3 A 48-year-old man previously in good health presents with an enlarging mole over his right foot's
dorsum. A biopsy shows melanoma, 1.1 mm in thickness, with ulceration, 2 mitoses per mm^2, with
a positive posterior margin. Wide local excision shows no residual melanoma. Sentinel inguinal
lymph node biopsy is negative. On a follow-up visit 6 months later, a 0.5 cm subcutaneous nodule is
noted over the right shin. No other skin lesions are noted, and there is no palpable inguinal or other
lymphadenopathy on the examination. The biopsy of the subcutaneous nodule shows melanoma.
No BRAF mutation is identified. FDG-PET/CT from top of the skull to toes shows a hypermetabolic
subcutaneous nodule over the right shin, with no other evidence of metastatic disease. MRI of the
brain is negative for intracranial metastasis. The patient's ECOG performance status is 0.
Hematologic and chemistry profiles are within normal limits. 

Which of the following is the most appropriate management strategy?

 Complete surgical excision of the subcutaneous nodule, followed by 1 year of nivolumab

 Complete surgical excision of the subcutaneous nodule, followed by nivolumab until progression

 Nivolumab until progression

 Complete surgical excision and complete inguinal lymph node dissection

Submit
Submit
Question 3 of 37

3 A 48-year-old man previously in good health presents with an enlarging mole over his right foot's
dorsum. A biopsy shows melanoma, 1.1 mm in thickness, with ulceration, 2 mitoses per mm^2, with
a positive posterior margin. Wide local excision shows no residual melanoma. Sentinel inguinal
lymph node biopsy is negative. On a follow-up visit 6 months later, a 0.5 cm subcutaneous nodule is
noted over the right shin. No other skin lesions are noted, and there is no palpable inguinal or other
lymphadenopathy on the examination. The biopsy of the subcutaneous nodule shows melanoma.
No BRAF mutation is identified. FDG-PET/CT from top of the skull to toes shows a hypermetabolic
subcutaneous nodule over the right shin, with no other evidence of metastatic disease. MRI of the
brain is negative for intracranial metastasis. The patient's ECOG performance status is 0.
Hematologic and chemistry profiles are within normal limits. 

Which of the following is the most appropriate management strategy?

 Complete surgical excision of the subcutaneous nodule, followed by 1 year of nivolumab

 Complete surgical excision of the subcutaneous nodule, followed by nivolumab until progression

 Nivolumab until progression

 Complete surgical excision and complete inguinal lymph node dissection

Rationale:
This patient with initially stage IIA melanoma (T2b N0) of the right foot, currently with evidence of a
single in-transit metastasis, which now represents stage IIIB disease. As there is no other evidence
of metastatic disease, and the isolated in-transit metastasis appears to be resectable, the
recommended primary treatment in this setting is complete surgical excision of the in-transit to clear
margins. Following excision, recommended options include observation and adjuvant therapy. It
would be reasonable to offer adjuvant immunotherapy for one year in this case. Complete lymph
node dissection is not indicated in the absence of clinical nodal involvement.

References:
Read RL, Thompson JF. Managing in-transit melanoma metastases in the new era of effective
systemic therapies for melanoma. Expert Rev Clin Pharmacol. 2019;12(12):1107-1119.
https://pubmed.ncbi.nlm.nih.gov/31687857/

Next
Question 4 of 37

4 A 26-year-old woman presents with an enlarging mole on her right upper back. Biopsy shows
melanoma, 0.5 mm thick, without ulceration, 1 mitosis per mm^2, with close margins. No palpable
lymphadenopathy is noted on exam. She undergoes a wide local excision, which shows no residual
melanoma. She has a history of a previous early-stage melanoma excised two years ago from her
left arm. She reports a history of melanoma in her mother and two of her brothers, a history of
breast cancer in an aunt (non-blood relative), and pancreatic cancer in a maternal uncle.

Which of the following is the most commonly identified mutated gene in familial forms of melanoma?

 BAP1

 BRCA1

 BRCA2

 CDKN2A

Submit
Submit
Question 4 of 37

4 A 26-year-old woman presents with an enlarging mole on her right upper back. Biopsy shows
melanoma, 0.5 mm thick, without ulceration, 1 mitosis per mm^2, with close margins. No palpable
lymphadenopathy is noted on exam. She undergoes a wide local excision, which shows no residual
melanoma. She has a history of a previous early-stage melanoma excised two years ago from her
left arm. She reports a history of melanoma in her mother and two of her brothers, a history of
breast cancer in an aunt (non-blood relative), and pancreatic cancer in a maternal uncle.

Which of the following is the most commonly identified mutated gene in familial forms of melanoma?

 BAP1

 BRCA1

 BRCA2

 CDKN2A

Rationale:
CDKN2A is the most commonly identified mutated gene in familial forms of melanoma. This gene
encodes two proteins, p16, and p14ARF; p16 is the more commonly impaired protein. The
prevalence of these mutations in families with hereditary melanoma is highly variable and ranges
from 20% to 60% in certain families, depending on geographic location. The prevalence of
CDKN2A germline mutations in the general population is much lower, estimated at around 1.2% in
individuals with a single primary melanoma and 2.9% in individuals with multiple primary
melanomas. A weaker association between BRCA1 and BRCA2 mutations and melanoma has
been found in some but not all studies. BAP1 germline mutations have been identified in families
with cutaneous and uveal melanoma.

References:
Goldstein AM, Chan M, Harland M, et al. High-risk melanoma susceptibility genes and pancreatic
cancer, neural system tumors, and uveal melanoma across GenoMEL. Cancer Res.
2006;66(20):9818-9828. https://pubmed.ncbi.nlm.nih.gov/17047042/

Next
Question 5 of 37

5 A 43-year-old woman presents to her dermatologist with a pigmented lesion on the abdomen.
Physical examination reveals an ulcerated neoplasm with concerning features on dermoscopy. A
shave biopsy reveals a 5.70 mm Breslow, Clarks level V, ulcerated, nodular melanoma with 5
mitoses/mm^2, moderate tumor-infiltrating lymphocytes (TIL) no evidence of satellitosis or
regression. There is the presence of Angio-lymphatic, and perineural invasion and margins are
involved. A  wide local excision (WLE)  and sentinel lymph node biopsy are performed and tested.
Two lymph nodes are positive for melanoma, and the largest metastatic deposit is 4 mm. 

Which of the following mutations is most likely to be found on genetic analysis of the primary
specimen?

 GNA11 c.547C > T (R183C) mutation

 BRAF c.1799T > A (V600E) mutation

 PIK3CA c.1624G > A (E542K) mutation

Submit
Submit
Question 5 of 37

5 A 43-year-old woman presents to her dermatologist with a pigmented lesion on the abdomen.
Physical examination reveals an ulcerated neoplasm with concerning features on dermoscopy. A
shave biopsy reveals a 5.70 mm Breslow, Clarks level V, ulcerated, nodular melanoma with 5
mitoses/mm^2, moderate tumor-infiltrating lymphocytes (TIL) no evidence of satellitosis or
regression. There is the presence of Angio-lymphatic, and perineural invasion and margins are
involved. A  wide local excision (WLE)  and sentinel lymph node biopsy are performed and tested.
Two lymph nodes are positive for melanoma, and the largest metastatic deposit is 4 mm. 

Which of the following mutations is most likely to be found on genetic analysis of the primary
specimen?

 GNA11 c.547C > T (R183C) mutation

 BRAF c.1799T > A (V600E) mutation

 PIK3CA c.1624G > A (E542K) mutation

Rationale:
BRAF exon 15 (37 to 50%), NRAS codon 61 (13 to 25%), and NF1 (12%) mutations account for the
majority of mutations found in cutaneous melanoma. Based on its association with chronically sun-
damaged (CSD) skin, cutaneous melanoma can be categorized as CSD or non-CSD melanoma.
CSD and non-CSD melanoma have different clinicopathological characteristics and are associated
with different driver mutations. CSD melanomas typically arise in older (> 55 years) patients on sun-
exposed areas (head/neck, dorsal surfaces of distal extremities). They are associated with specific
driver mutations (BRAF non-V600E, NRAS, NF1, or KIT) and genetic signatures of UV-induced
DNA damage [G > T (UVA) or C > T (UVB)] transitions. In contrast, non-CSD melanomas typically
arise in younger (< 55 years) patients on intermittently sun-exposed areas (trunk, proximal
extremities), and are associated with BRAF V600E/K driver mutations, and often lack the genetic
signature of UV mutagenesis. PIK3CA exon 9 mutations are observed in up to 26% of breast
cancer. Although PTEN loss is seen in 7 to 28% of melanoma, these mutations typically occur in
exon 5 rather than exon 9.

References:
Curtin JA, Fridlyand J, Kageshita T, et al. Distinct sets of genetic alterations in melanoma. N Engl J
Med. 2005;353(20):2135-2147. https://pubmed.ncbi.nlm.nih.gov/16291983/
Hodis E, Watson IR, Kryukov GV, et al. A landscape of driver mutations in melanoma. Cell.
2012;150(2):251-263. https://pubmed.ncbi.nlm.nih.gov/22817889/
Cancer Genome Atlas Network. Genomic Classification of Cutaneous Melanoma. Cell.
2015;161(7):1681-1696. https://pubmed.ncbi.nlm.nih.gov/26091043/

Next
Question 6 of 37

6 A 61-year-old man presents with floaters in his right eye, which appeared three weeks ago. On
fundoscopic and ultrasonographic examination, he is found to have a right choroidal pigmented
lesion. Upon further evaluation, it is determined to be a uveal melanoma.

Which of the following is the most commonly identified somatic mutation in this tumor type?

 GNAQ

 BRAF

 NRAS

 KIT

Submit
Submit
Question 6 of 37

6 A 61-year-old man presents with floaters in his right eye, which appeared three weeks ago. On
fundoscopic and ultrasonographic examination, he is found to have a right choroidal pigmented
lesion. Upon further evaluation, it is determined to be a uveal melanoma.

Which of the following is the most commonly identified somatic mutation in this tumor type?

 GNAQ

 BRAF

 NRAS

 KIT

Rationale:
GNAQ and GNA11 are the most commonly identified somatic mutations in uveal melanoma, with a
mutation in either gene occurring in about 80% of cases. Other commonly identified mutations in
this tumor type include BAP1, SF3B1, and EIF1AX. Unlike in cutaneous melanoma, mutations in
BRAF, NRAS, and KIT are rarely found in uveal melanoma.

References:
Van Raamsdonk CD, Bezrookove V, Green G, et al. Frequent somatic mutations of GNAQ in uveal
melanoma and blue naevi. Nature. 2009;457(7229):599-602.
https://pubmed.ncbi.nlm.nih.gov/19078957/

Next
Question 7 of 37

7 A 45-year-old man presents with floaters in his left eye, which appeared 4 days ago. On
fundoscopic and ultrasonographic evaluation, he is found to have a left ciliary body pigmented
lesion. Upon further evaluation, it is determined to be a primary ocular melanoma. He reports a
family history of cutaneous melanoma in two of his younger sisters, mesothelioma in his mother,
and renal cell carcinoma in a maternal uncle. 

A germline mutation in which of the following genes is most likely to be found in this patient?

 BRAF

 BAP1

 GNA11

 BRCA1

Submit
Submit
Question 7 of 37

7 A 45-year-old man presents with floaters in his left eye, which appeared 4 days ago. On
fundoscopic and ultrasonographic evaluation, he is found to have a left ciliary body pigmented
lesion. Upon further evaluation, it is determined to be a primary ocular melanoma. He reports a
family history of cutaneous melanoma in two of his younger sisters, mesothelioma in his mother,
and renal cell carcinoma in a maternal uncle. 

A germline mutation in which of the following genes is most likely to be found in this patient?

 BRAF

 BAP1

 GNA11

 BRCA1

Rationale:
Germline mutations in the BRCA1-associated gene 1 (BAP1) have been identified in families with
cutaneous and uveal melanoma and linked to a BAP-1 tumor predisposition syndrome. The risk of
developing mesothelioma and renal cell carcinoma also appears to be increased. GNA11 and
GNAQ somatic mutations are commonly identified in uveal melanoma; germline mutations in these
genes are not commonly identified. Somatic mutations in BRAF are commonly present in
cutaneous melanoma. 

References:
Njauw CN, Kim I, Piris A, et al. Germline BAP1 inactivation is preferentially associated with
metastatic ocular melanoma and cutaneous-ocular melanoma families. PLoS One.
2012;7(4):e35295. https://pubmed.ncbi.nlm.nih.gov/22545102/
Testa JR, Cheung M, Pei J, et al. Germline BAP1 mutations predispose to malignant
mesothelioma. Nat Genet. 2011;43(10):1022-1025. https://pubmed.ncbi.nlm.nih.gov/21874000/

Next
Question 8 of 37

8 A 36-year-old woman presents with an enlarging pigmented lesion on her right arm. Biopsy shows a
nodular melanoma, 2.5 mm in depth, 3 mitoses per mm^2, with ulceration and positive margins.
Wide local excision and axillary sentinel lymph node biopsy are performed. There is no residual
disease on the wide local excision sample. The sentinel node shows a 1.5 mm focus of metastatic
melanoma. Molecular testing shows no BRAF mutation. Tumor PD-L1 expression is found to be 1%.
PET/CT and MRI of the brain are negative for distant metastatic disease.

Which of the following is the best next step in management?

 Lymph node dissection

 Ipilimumab

 Nivolumab

 Dabrafenib and trametinib

Submit
Submit
Question 8 of 37

8 A 36-year-old woman presents with an enlarging pigmented lesion on her right arm. Biopsy shows a
nodular melanoma, 2.5 mm in depth, 3 mitoses per mm^2, with ulceration and positive margins.
Wide local excision and axillary sentinel lymph node biopsy are performed. There is no residual
disease on the wide local excision sample. The sentinel node shows a 1.5 mm focus of metastatic
melanoma. Molecular testing shows no BRAF mutation. Tumor PD-L1 expression is found to be 1%.
PET/CT and MRI of the brain are negative for distant metastatic disease.

Which of the following is the best next step in management?

 Lymph node dissection

 Ipilimumab

 Nivolumab

 Dabrafenib and trametinib

Rationale:
This is a patient with stage IIIB melanoma. Immediate completion lymph node dissection is no
longer recommended following a positive sentinel lymph node biopsy; periodic monitoring of the
nodal basin is performed instead to evaluate for recurrence. Adjuvant therapy would generally be
recommended, or at least considered, for patients with stage IIIB melanoma. Nivolumab has been
shown to significantly improve recurrence-free survival in the adjuvant setting compared to
ipilimumab regardless of PD-L1 expression, with a lower rate of grade 3 and 4 adverse events. The
combination of dabrafenib and trametinib would be an option for patients whose tumor harbors a
BRAF V600 activating mutation, which is not the case in this patient.

References:
Faries MB, Thompson JF, Cochran AJ, et al. Completion Dissection or Observation for Sentinel-
Node Metastasis in Melanoma. New England Journal of Medicine. 2017;376(23):2211-2222.
https://www.nejm.org/doi/full/10.1056/nejmoa1613210
Weber J, Mandala M, Del Vecchio M, et al. Adjuvant Nivolumab versus Ipilimumab in Resected
Stage III or IV Melanoma. New England Journal of Medicine. 2017;377(19):1824-1835.
https://www.nejm.org/doi/full/10.1056/NEJMoa1709030

Next
Question 9 of 37

9 A 52-year-old man with no previous medical history presents with a new pigmented lesion on his left
arm. Biopsy shows melanoma, 1.1 mm in thickness, with ulceration, 1 mitosis per mm^2. BRAF
testing is performed and shows a BRAF V600E mutation. Wide local excision shows no residual
melanoma. Sentinel axillary lymph node biopsy is negative for melanoma. His ECOG performance
status is 0. Hematologic and chemistry profiles are within normal limits.

Which of the following is the next best step in management?

 Observation

 Adjuvant dabrafenib and trametinib

 Adjuvant ipilimumab and nivolumab

 Adjuvant interferon alpha

Submit
Submit
Question 9 of 37

9 A 52-year-old man with no previous medical history presents with a new pigmented lesion on his left
arm. Biopsy shows melanoma, 1.1 mm in thickness, with ulceration, 1 mitosis per mm^2. BRAF
testing is performed and shows a BRAF V600E mutation. Wide local excision shows no residual
melanoma. Sentinel axillary lymph node biopsy is negative for melanoma. His ECOG performance
status is 0. Hematologic and chemistry profiles are within normal limits.

Which of the following is the next best step in management?

 Observation

 Adjuvant dabrafenib and trametinib

 Adjuvant ipilimumab and nivolumab

 Adjuvant interferon alpha

Rationale:
This is a patient with stage IIA melanoma (T2b N0). No adjuvant therapy is indicated at this time,
except in the context of a clinical trial.

References:
Kaufman HL, Kirkwood JM, Hodi FS, et al. The Society for Immunotherapy of Cancer consensus
statement on tumour immunotherapy for the treatment of cutaneous melanoma. Nat Rev Clin
Oncol. 2013;10(10):588-598. https://pubmed.ncbi.nlm.nih.gov/23982524/

Next
Question 10 of 37

10 A 65-year-old man presents in the clinic with unintentional weight loss, palpable right axillary lymph
nodes, and fatigue. The patient has a history of stage IIA malignant melanoma of the back. Three
years ago, a wide local excision and sentinel lymph node biopsy was completed. ECOG
performance status is 1. Cross-sectional imaging of the chest, abdomen, and pelvis demonstrates
numerous nodules in the bilateral lung and liver and right axillary adenopathy suspicious for
metastatic disease. Biopsy of a right axillary lymph node reveals metastatic melanoma. Tests show
BRAF wildtype. Laboratory studies are unremarkable except for lactate dehydrogenase (LDH) of
500 U/L.

Based on this patient‘s diagnosis and tumor characteristics, what is the most appropriate option for
first-line treatment?

 Nivolumab with ipilimumab

 Ipilimumab

 Encorafenib with binimetinib

 High-dose interleukin-2 (IL-2)

Submit
Submit
Question 10 of 37

10 A 65-year-old man presents in the clinic with unintentional weight loss, palpable right axillary lymph
nodes, and fatigue. The patient has a history of stage IIA malignant melanoma of the back. Three
years ago, a wide local excision and sentinel lymph node biopsy was completed. ECOG
performance status is 1. Cross-sectional imaging of the chest, abdomen, and pelvis demonstrates
numerous nodules in the bilateral lung and liver and right axillary adenopathy suspicious for
metastatic disease. Biopsy of a right axillary lymph node reveals metastatic melanoma. Tests show
BRAF wildtype. Laboratory studies are unremarkable except for lactate dehydrogenase (LDH) of
500 U/L.

Based on this patient‘s diagnosis and tumor characteristics, what is the most appropriate option for
first-line treatment?

 Nivolumab with ipilimumab

 Ipilimumab

 Encorafenib with binimetinib

 High-dose interleukin-2 (IL-2)

Rationale:
In patients with a good performance status without a BRAF V600 activating mutation, the best initial
choice for treatment is a combination (PD-1 inhibitor/CTLA-4 inhibitor) or single-agent PD-1
inhibitor therapy (Choice A). In this case, the patient‘s tumor does not have a BRAF V600E
activating mutation, which precludes choice C. The combination of PD-1 inhibitor therapy and
CTLA-4 inhibitor therapy showed overall survival (OS) superiority compared with either agent alone
in CheckMate 067 (Choice B). High-dose Interleukin 2 was the first treatment for metastatic
melanoma with a median OS of 11.4 months. However, given the improved responses with new
agents and higher toxicity profile of high dose IL2, it is now a less preferred approach (choice D).

References:
Hodi FS, Chiarion-Sileni V, Gonzalez R, et al. Nivolumab plus ipilimumab or nivolumab alone
versus ipilimumab alone in advanced melanoma (CheckMate 067): 4-year outcomes of a
multicentre, randomized, phase 3 trial [published correction appears in Lancet Oncol. 2018
Dec;19(12):e668] [published correction appears in Lancet Oncol. 2018 Nov;19(11):e581].
Lancet Oncol. 2018;19(11):1480-1492. doi:10.1016/S1470-2045(18)30700-9
Weber JS, Gibney G, Sullivan RJ, et al. Sequential administration of nivolumab and
ipilimumab with a planned switch in patients with advanced melanoma (CheckMate 064): an
open-label, randomized, phase 2 trial [published correction appears in Lancet Oncol. 2016
Jul;17 (7):e270]. Lancet Oncol. 2016;17(7):943-955. doi:10.1016/S1470-2045(16)30126-7

Next
Question 11 of 37

11 A 51-year-old woman with a history of melanoma of the abdomen approximately 15 years ago
treated with wide local excision presents a palpable nodule on her right flank. Excision is performed,
and the pathology is consistent with a 2-cm deposit of metastatic melanoma in the subcutaneous
tissue; margins are negative. A staging PET/CT is negative for other sites of disease. Molecular
testing is negative for the presence of a BRAF V600 mutation.

Which of the following is the most appropriate next step?

 Close monitoring with clinical and radiographic imaging

 Adjuvant therapy for one year with pembrolizumab

 Adjuvant therapy for one year with ipilimumab

 Adjuvant therapy for one year with Interferon Alfa-2a

Submit
Submit
Question 11 of 37

11 A 51-year-old woman with a history of melanoma of the abdomen approximately 15 years ago
treated with wide local excision presents a palpable nodule on her right flank. Excision is performed,
and the pathology is consistent with a 2-cm deposit of metastatic melanoma in the subcutaneous
tissue; margins are negative. A staging PET/CT is negative for other sites of disease. Molecular
testing is negative for the presence of a BRAF V600 mutation.

Which of the following is the most appropriate next step?

 Close monitoring with clinical and radiographic imaging

 Adjuvant therapy for one year with pembrolizumab

 Adjuvant therapy for one year with ipilimumab

 Adjuvant therapy for one year with Interferon Alfa-2a

Rationale:
Adjuvant therapy with nivolumab has demonstrated an improvement in recurrence-free survival
(RFS) compared to ipilimumab; additionally, PD-1 based therapy has an improved tolerability
profile. Pembrolizumab has also demonstrated an improved RFS benefit compared to placebo.
Targeted therapy with adjuvant dabrafenib and trametinib is an option for patients whose tumors
harbor a BRAFV600 mutation, but this patient‘s tumor is negative for BRAFV600.

References:
Ascierto PA, Del Vecchio M, Mandalà M, et al. Adjuvant nivolumab versus ipilimumab in
resected stage IIIB-C and stage IV melanoma (CheckMate 238): 4-year results from a
multicentre, double-blind, randomised, controlled, phase 3 trial. Lancet Oncol.
2020;21(11):1465-1477. doi:10.1016/S1470-2045(20)30494-0
Eggermont AMM, Blank CU, Mandala M, et al. Longer Follow-Up Confirms Recurrence-Free
Survival Benefit of Adjuvant Pembrolizumab in High-Risk Stage III Melanoma: Updated
Results From the EORTC 1325-MG/KEYNOTE-054 Trial. J Clin Oncol. 2020;38(33):3925-3936.
doi:10.1200/JCO.20.02110

Next
Question 12 of 37

12 A 27-year-old woman presented to a dermatologist for a routine skin examination. The


dermatologist noticed a lesion with scalloped borders on her right arm.

Which additional features would be alarming?

 There is no bleeding, itching, or pain.

 The lesion is flesh colored.

 The lesion has changed over time.

 She has other surrounding lesions.

Submit
Submit
Question 12 of 37

12 A 27-year-old woman presented to a dermatologist for a routine skin examination. The


dermatologist noticed a lesion with scalloped borders on her right arm.

Which additional features would be alarming?

 There is no bleeding, itching, or pain.

 The lesion is flesh colored.

 The lesion has changed over time.

 She has other surrounding lesions.

Rationale:
Certain features in skin lesions are associated with an increased risk of a malignant melanoma
diagnosis. Change in a pre-existing lesion is very alarming. Therefore, even in a lesion that may
look concerning, stability and lack of change in its appearance over time are less suggestive for
developing melanoma. Bleeding, itching, or pain associated with a lesion are also very concerning
for both a diagnosis of melanoma and more advanced melanoma; therefore, the absence of these
is less concerning. Melanomas can lack pigment, so all changing lesions, regardless of color,
should be evaluated and biopsied, and lack of pigment itself is not a concerning feature.
Surrounding lesions can be common in patients, and a lesion that does not look like the others (the
ugly duckling) or stands out is concerning.

References:
Fears TR, Guerry Dt, Pfeiffer RM, et al. Identifying individuals at high risk of melanoma: a
practical predictor of absolute risk. J Clin Oncol. 2006;24(22):3590-3596

Next
Question 13 of 37

13 A 75-year-old man presented to a dermatologist after noting multiple pigmented nodules on his
scalp. Three years prior, he was diagnosed with a 2.60-mm Breslow, Clark level III, focally
ulcerated, and superficial spreading melanoma. Additionally, testing found 1 mitosis/mm^2, a partial
regression, moderate tumor-infiltrating lymphocytes (TIL) with no evidence of satellitosis,
angiolymphatic, or perineural invasion. A sentinel lymph node biopsy was negative at that time.
Today a shave biopsy is performed on one of the nodules, and the pathology notes malignant
melanoma, with no overlying connection to the epidermis, consistent with a dermal metastasis.

Which of the following mutations is most likely to be found on genetic analysis of the primary
specimen?

 GNA11 c.547C>T (R183C) mutation

 NRAS c.38G>T (G13V) mutation

 KRAS c.34G>T (G12C) mutation

 PIK3CA c.1624G>A (E542K) mutation

Submit
Submit
Question 13 of 37

13 A 75-year-old man presented to a dermatologist after noting multiple pigmented nodules on his
scalp. Three years prior, he was diagnosed with a 2.60-mm Breslow, Clark level III, focally
ulcerated, and superficial spreading melanoma. Additionally, testing found 1 mitosis/mm^2, a partial
regression, moderate tumor-infiltrating lymphocytes (TIL) with no evidence of satellitosis,
angiolymphatic, or perineural invasion. A sentinel lymph node biopsy was negative at that time.
Today a shave biopsy is performed on one of the nodules, and the pathology notes malignant
melanoma, with no overlying connection to the epidermis, consistent with a dermal metastasis.

Which of the following mutations is most likely to be found on genetic analysis of the primary
specimen?

 GNA11 c.547C>T (R183C) mutation

 NRAS c.38G>T (G13V) mutation

 KRAS c.34G>T (G12C) mutation

 PIK3CA c.1624G>A (E542K) mutation

Rationale:
BRAF exon 15 (37 to 50%), NRAS codon 61 (13 to 25%), and NF1 (11.9%) mutations account for
the majority of mutations found in cutaneous melanoma. Based on its association with chronically
sun-damaged (CSD) skin, cutaneous melanoma can be categorized as CSD or non-CSD
melanoma. CSD and non-CSD melanoma have different clinicopathological characteristics and are
associated with different driver mutations. CSD melanomas typically arise in older (>55 years)
patients on sun-exposed areas (head/neck, dorsal surfaces of distal extremities). They are
associated with specific driver mutations (BRAFnonV600E, NRAS, NF1, or KIT) and genetic
signatures of UV-induced DNA damage [G > T (UVA) or C > T (UVB)] transitions. In contrast, non-
CSD melanomas typically arise in younger (<55 years) patients on intermittently sun-exposed areas
(trunk, proximal extremities), and are associated with BRAFV600E/K driver mutations and often
lack genetic signature of UV mutagenesis. Mutations in GNA11 and GNAQ are commonly observed
in uveal melanoma but not in cutaneous melanoma. KRAS codon 12 mutations are commonly (15
to 25%) observed in lung cancer but not in melanoma. PIK3CA exon nine mutations are observed
in up to 26% of breast cancer. Although PTEN loss is seen in 7 to 28% of melanoma, these
mutations typically occur in exon 5 rather than exon 9.

References:
Curtin JA, Fridlyand J, Kageshita T, et al. Distinct Sets of Genetic Alterations in Melanoma.
New England Journal of Medicine. 2005;353(20):2135-2147
Hodis E, Watson IR, Kryukov GV, et al. A landscape of driver mutations in melanoma. Cell.
2012;150(2):251-263

Next
Question 14 of 37

14 A 64-year-old woman presented with a pigmented and ulcerated lesion on her left leg and palpable
left inguinal lymph nodes. She underwent wide local excision of the lesion with complete lymph
node dissection of the left groin. Final pathology showed a 3-mm thick ulcerated lesion, and two of
10 lymph nodes were positive for melanoma with no extracapsular extension. The largest tumor
deposit in the lymph nodes was 7 mm. BRAF mutation was negative. She is now healed and ready
for additional treatment if needed.

Which of the following should you recommend now?

 Immunotherapy, followed by radiation therapy to the groin

 Radiation therapy to the left side of the groin, followed by immunotherapy

 Radiation therapy to the left side of the groin and primary site, followed by immunotherapy

 Immunotherapy

Submit
Submit
Question 14 of 37

14 A 64-year-old woman presented with a pigmented and ulcerated lesion on her left leg and palpable
left inguinal lymph nodes. She underwent wide local excision of the lesion with complete lymph
node dissection of the left groin. Final pathology showed a 3-mm thick ulcerated lesion, and two of
10 lymph nodes were positive for melanoma with no extracapsular extension. The largest tumor
deposit in the lymph nodes was 7 mm. BRAF mutation was negative. She is now healed and ready
for additional treatment if needed.

Which of the following should you recommend now?

 Immunotherapy, followed by radiation therapy to the groin

 Radiation therapy to the left side of the groin, followed by immunotherapy

 Radiation therapy to the left side of the groin and primary site, followed by immunotherapy

 Immunotherapy

Rationale:
The only appropriate treatment for this patient is immunotherapy. Radiation therapy is not indicated
after complete lymph node dissection unless there are high-risk features for locoregional
recurrence (multiple lymph nodes involved, matted lymph nodes, and extracapsular extension).
Radiation to the groin or the primary site is not considered appropriate.

References:
Henderson MA, Burmeister BH, Ainslie J, et al. Adjuvant lymph-node field radiotherapy
versus observation only in patients with melanoma at high risk of further lymph-node field
relapse after lymphadenectomy (ANZMTG 01.02/TROG 02.01): 6-year follow-up of a phase 3,
randomised controlled trial. Lancet Oncol. 2015;16(9):1049-1060. doi:10.1016/S1470-
2045(15)00187-4
Daniels CP, Liu HY, Porceddu SV. Indications and limits of postoperative radiotherapy for
skin malignancies. Curr Opin Otolaryngol Head Neck Surg. 2021;29(2):100-106.
doi:10.1097/MOO.0000000000000696

Next
Question 15 of 37

15 A 56-year-old woman was seen after wide local excision and sentinel lymph node biopsy of a dark
ulcerated lesion on her back. Final pathology showed malignant melanoma 4.5 mm in depth with
negative margins. The sentinel lymph node biopsy from her left axillary area was negative. BRAF
mutation was positive. Her disease stage grouping was stage IIC. Clinically she has healed well and
is ready to resume work. She would like to know if there is anything else that she can do to improve
her chances of preventing cancer recurrence.

Which of the following should you recommend now?

 High-dose interferon

 Pembrolizumab

 Dabrafenib and trametinib

 Observation

Submit
Submit
Question 15 of 37

15 A 56-year-old woman was seen after wide local excision and sentinel lymph node biopsy of a dark
ulcerated lesion on her back. Final pathology showed malignant melanoma 4.5 mm in depth with
negative margins. The sentinel lymph node biopsy from her left axillary area was negative. BRAF
mutation was positive. Her disease stage grouping was stage IIC. Clinically she has healed well and
is ready to resume work. She would like to know if there is anything else that she can do to improve
her chances of preventing cancer recurrence.

Which of the following should you recommend now?

 High-dose interferon

 Pembrolizumab

 Dabrafenib and trametinib

 Observation

Rationale:
Up until recently, the only approved adjuvant treatment for this stage was high-dose interferon.
Recently, the completion of KEYNOTE-716 changed the landscape. This randomized controlled
trial demonstrated a statistically significant improvement in recurrence-free survival (RFS) at the
time of the first interim analysis for patients randomly selected to the pembrolizumab arm compared
with placebo, with a hazard ratio of 0.65 (95% CI 0.46-0.92; P = 0.0132). The median RFS was not
reached in either arm. This study led to the approval of pembrolizumab in this indication. High-dose
interferon used to be the standard of care but has rarely been used due to its toxicity. Dabrafenib
and trametinib have not been studied in this indication. Observation is still an option for patients
who decline to receive pembrolizumab. This patient would like to decrease her risk of disease
recurrence, hence observation is not appropriate for her.

References:
Luke JJ, Rutkowski P, Queirolo P, et al. LBA3 Pembrolizumab versus placebo after complete
resection of high-risk stage II melanoma: Efficacy and safety results from the KEYNOTE-716
double-blind phase III trial. Ann Oncol. 2021;32 (suppl_5): S1314-S1315. doi:
10.1016/j.annonc.2021.08.2116
Poklepovic AS, Luke JJ. Considering adjuvant therapy for stage II melanoma. Cancer.
2020;126(6):1166-1174. doi: 10.1002/cncr.32585
Next
Question 16 of 37

16 A 45-year-old woman presents with generalized seizures and was found to have a solitary right
frontal lobe mass with perilesional edema and midline shift. She had reported progressively
worsening headaches over the past six weeks. Her medical history consists of a stage II BRAF-
mutated melanoma that was resected from her back a year ago. At that time, she had a wide local
excision and sentinel lymph node biopsy of the right axilla. Antiseizure therapy and dexamethasone
were started.

Which of the following is the most appropriate next step?

 Dabrafenib and trametinib

 Surgical resection

 Ipilimumab and nivolumab

 Temozolomide

Submit
Submit
Question 16 of 37

16 A 45-year-old woman presents with generalized seizures and was found to have a solitary right
frontal lobe mass with perilesional edema and midline shift. She had reported progressively
worsening headaches over the past six weeks. Her medical history consists of a stage II BRAF-
mutated melanoma that was resected from her back a year ago. At that time, she had a wide local
excision and sentinel lymph node biopsy of the right axilla. Antiseizure therapy and dexamethasone
were started.

Which of the following is the most appropriate next step?

 Dabrafenib and trametinib

 Surgical resection

 Ipilimumab and nivolumab

 Temozolomide

Rationale:
Upfront surgical resection or radiation therapy for symptomatic brain metastases is the standard of
care. Immunotherapy or targeted therapy can be used upfront only in small (3 cm) asymptomatic
brain metastases with close monitoring for early detection of progressing brain metastasis, which
would require surgery or radiation therapy. Temozolomide has no role in treating melanoma brain
metastasis.

References:
Long GV, Atkinson V, Lo S, et al. Combination nivolumab and ipilimumab or nivolumab
alone in melanoma brain metastases: a multicentre randomised phase 2 study. Lancet
Oncol. 2018;19(5):672-681. doi:10.1016/S1470-2045(18)30139-6
Tawbi HA, Forsyth PA, Hodi FS, et al. Long-term outcomes of patients with active melanoma
brain metastases treated with combination nivolumab plus ipilimumab (CheckMate 204):
final results of an open-label, multicentre, phase 2 study. Lancet Oncol. 2021;22(12):1692-
1704. doi:10.1016/S1470-2045(21)00545-3

Next
Question 17 of 37

17 A 57-year-old woman is referred to medical oncology for evaluation of metastatic cancer. She has a
history of a right ocular (uveal) melanoma that was treated with removal of the right eye
(enucleation) five years ago. Surveillance imaging showed multiple small hypodense lesions in both
liver lobes, concerning for metastasis. She is asymptomatic. Physical examination shows no
hepatomegaly or abdominal tenderness. Laboratory test results including complete blood count and
liver function tests are within normal limits. Liver biopsy confirms metastatic melanoma. Mutational
profiling of the tumor shows mutations in GNAQ and BAP1 genes, indicating melanoma of uveal
origin. Additional testing reveals the patient to be positive for the HLA-A*02:01 allele.

Which of the following treatment options is most appropriate for this patient?

 Tebentafusp

 Pembrolizumab

 Ipilimumab

 Dacarbazine

Submit
Submit
Question 17 of 37

17 A 57-year-old woman is referred to medical oncology for evaluation of metastatic cancer. She has a
history of a right ocular (uveal) melanoma that was treated with removal of the right eye
(enucleation) five years ago. Surveillance imaging showed multiple small hypodense lesions in both
liver lobes, concerning for metastasis. She is asymptomatic. Physical examination shows no
hepatomegaly or abdominal tenderness. Laboratory test results including complete blood count and
liver function tests are within normal limits. Liver biopsy confirms metastatic melanoma. Mutational
profiling of the tumor shows mutations in GNAQ and BAP1 genes, indicating melanoma of uveal
origin. Additional testing reveals the patient to be positive for the HLA-A*02:01 allele.

Which of the following treatment options is most appropriate for this patient?

 Tebentafusp

 Pembrolizumab

 Ipilimumab

 Dacarbazine

Rationale:
Uveal melanoma has a distinct disease biology, as compared to cutaneous melanoma. Mutations in
the GNAQ or GNA11 genes are most common in primary uveal melanoma, with subsequent
mutations in the BAP1 or SF3B1 genes associated with metastasis. Uveal melanoma does not
have mutations in the BRAF V600 genes that can be therapeutically targeted in cutaneous
melanoma. Metastatic uveal melanoma is also poorly responsive to immune checkpoint inhibitor
therapy. Tebentafusp (IMCgp100) is a novel, bispecific protein that binds the CD3 receptor on T
cells and engages gp100 (a protein expressed by normal melanocytes and melanoma cells) on
tumor cells. The expression of the HLA allele, HLA-A*02:01, is necessary for presentation of gp100
on the cell surface. In a randomized phase 3 study, patients with metastatic uveal melanoma were
randomly selected to receive tebentafusp versus investigators‘ choice of therapy (pembrolizumab,
ipilimumab, or dacarbazine). Patients receiving tebentafusp demonstrated improved one-year
overall survival (73% versus 59%) and six-month progression-free survival (31% versus 19%),
leading to FDA approval of tebentafusp for metastatic uveal melanoma patients who are HLA-
A*02:01-positive. In the phase 3 trial by Nathan et al., tebentafusp was superior to pembrolizumab
or ipilimumab in patients with metastatic uveal melanoma. Cytotoxic chemotherapy has very limited
efficacy in metastatic uveal melanoma. In the phase 3 SUMIT trial, use of dacarbazine had a 0%
objective response rate.
References:
Nathan P, Hassel JC, Rutkowski P, et al. Overall Survival Benefit with Tebentafusp in
Metastatic Uveal Melanoma. N Engl J Med. 2021;385(13):1196-1206.
doi:10.1056/NEJMoa2103485
Algazi AP, Tsai KK, Shoushtari AN, et al. Clinical outcomes in metastatic uveal melanoma
treated with PD-1 and PD-L1 antibodies. Cancer. 2016;122(21):3344-3353.
doi:10.1002/cncr.30258
Carvajal RD, Piperno-Neumann S, Kapiteijn E, et al. Selumetinib in Combination With
Dacarbazine in Patients With Metastatic Uveal Melanoma: A Phase III, Multicenter,
Randomized Trial (SUMIT) [published correction appears in J Clin Oncol. 2018 Dec
10;36(35):3528]. J Clin Oncol. 2018;36(12):1232-1239. doi:10.1200/JCO.2017.74.1090

Next
Question 18 of 37

18 A 74-year-old man was referred to see you after being diagnosed with a neuroendocrine cancer on
his scalp. He had a flesh-colored lesion on his scalp that grew over the last three months; excisional
biopsy showed small, round, blue cells with sparse cytoplasm and abundant mitoses. Differential
diagnosis includes Merkel cell carcinoma or metastatic visceral neuroendocrine carcinoma to the
skin.

Which of the following immunohistochemistry results is most consistent with Merkel cell carcinoma?

 CK20-negative, TTF1 negative

 CK20-positive, TTF1 negative

 CK20-positive, TTF1-positive

 CK20-negative, TTF1-positive

Submit
Submit
Question 18 of 37

18 A 74-year-old man was referred to see you after being diagnosed with a neuroendocrine cancer on
his scalp. He had a flesh-colored lesion on his scalp that grew over the last three months; excisional
biopsy showed small, round, blue cells with sparse cytoplasm and abundant mitoses. Differential
diagnosis includes Merkel cell carcinoma or metastatic visceral neuroendocrine carcinoma to the
skin.

Which of the following immunohistochemistry results is most consistent with Merkel cell carcinoma?

 CK20-negative, TTF1 negative

 CK20-positive, TTF1 negative

 CK20-positive, TTF1-positive

 CK20-negative, TTF1-positive

Rationale:
It is difficult to differentiate between primary Merkel cell carcinoma and metastatic neuroendocrine
carcinoma (eg, metastatic small cell lung cancer). Immunohistochemistry is helpful in differentiating
between the two and informing a diagnosis of Merkel cell carcinoma. CK20 is positive in most
cases of Merkel cell carcinoma, whereas TTF1 is almost always negative in Merkel cell carcinoma.
TTF1 is positive in approximately 80% of small cell lung cancer cases. Metastatic small cell lung
cancer typically demonstrates CK20-negative and TTF1-positive histology in 80% of cases.

References:
Leech SN, Kolar AJ, Barrett PD, Sinclair SA, Leonard N. Merkel cell carcinoma can be
distinguished from metastatic small cell carcinoma using antibodies to cytokeratin 20 and
thyroid transcription factor 1. J Clin Pathol. 2001;54(9):727-729. doi:10.1136/jcp.54.9.727
Chan JK, Suster S, Wenig BM, Tsang WY, Chan JB, Lau AL. Cytokeratin 20 immunoreactivity
distinguishes Merkel cell (primary cutaneous neuroendocrine) carcinomas and salivary
gland small cell carcinomas from small cell carcinomas of various sites. Am J Surg Pathol.
1997;21(2):226-234. doi:10.1097/00000478-199702000-00014

Next
Question 19 of 37

19 A 56-year-old woman presented to the oncology clinic with a recently diagnosed a 1-mm deep,
ulcerated melanoma in the right thigh. She underwent primary resection and sentinel lymph node
resection revealing one (of one) positive lymph nodes with 1 mm of tumor involvement in the node.
PET-CT imaging showed no evidence of distant disease. She was followed with serial nodal
ultrasonography every three months. After one year, ultrasonogram demonstrated multiple
abnormal lymph nodes in the right side of her groin. Biopsy of a lymph node was consistent with
metastatic melanoma histologically similar to the original melanoma.

Which of the following should you do now?

 Pembrolizumab

 Complete lymph node dissection

 Nodal ultrasonogram every three months

 Clinical examination every three months

Submit
Submit
Question 19 of 37

19 A 56-year-old woman presented to the oncology clinic with a recently diagnosed a 1-mm deep,
ulcerated melanoma in the right thigh. She underwent primary resection and sentinel lymph node
resection revealing one (of one) positive lymph nodes with 1 mm of tumor involvement in the node.
PET-CT imaging showed no evidence of distant disease. She was followed with serial nodal
ultrasonography every three months. After one year, ultrasonogram demonstrated multiple
abnormal lymph nodes in the right side of her groin. Biopsy of a lymph node was consistent with
metastatic melanoma histologically similar to the original melanoma.

Which of the following should you do now?

 Pembrolizumab

 Complete lymph node dissection

 Nodal ultrasonogram every three months

 Clinical examination every three months

Rationale:
According to the MSLT II trial, full nodal dissection can be avoided in patients who are diagnosed
with localized melanoma that is clinically negative nodal with positive sentinel lymph nodes,
provided they receive adequate follow-up consisting of observation and nodal ultrasonography
every three months. The omission of full nodal dissection in patients with clinical surveillance
showed no negative impact on melanoma-specific survival (dissection arm: 86% +/- 1.3%;
observation arm: 86% +/- 1.2%) and lowered the risk for lymphedema from 21.4% to 6%. In the
MSLT II trial, complete lymph node dissection was reserved for recurrence of nodal melanoma
while on active surveillance. Pembrolizumab is approved for adjuvant treatment in patients who
have lymph node involvement after complete resection.

References:
Faries MB, Thompson JF, Cochran AJ, et al. Completion Dissection or Observation for
Sentinel-Node Metastasis in Melanoma. N Engl J Med. 2017;376(23):2211-2222.
doi:10.1056/NEJMoa1613210
Leiter U, Stadler R, Mauch C, et al. Final Analysis of DeCOG-SLT Trial: No Survival Benefit
for Complete Lymph Node Dissection in Patients With Melanoma With Positive Sentinel
Node. J Clin Oncol. 2019;37(32):3000-3008. doi:10.1200/JCO.18.02306
Next
Question 20 of 37

20 A 48-year-old man presents to the dermatology clinic with a mole on his right forearm that has been
painless but growing. He is otherwise feeling well and healthy. No other concerning skin lesions are
noted and physical examination findings are otherwise unremarkable. Biopsy of the mole shows
melanoma, 5.1 mm in depth, without ulceration. The patient undergoes a sentinel lymph node
biopsy, with two of three axillary lymph nodes showing involvement by melanoma. Molecular testing
shows wildtype BRAF.

Which of the following is the most appropriate next step?

 Observation with serial nodal ultrasonogram

 Completion lymph node dissection

 Serial nodal ultrasonogram and adjuvant anti-PD1 therapy

 Completion lymph node dissection and adjuvant immunotherapy

Submit
Submit
Question 20 of 37

20 A 48-year-old man presents to the dermatology clinic with a mole on his right forearm that has been
painless but growing. He is otherwise feeling well and healthy. No other concerning skin lesions are
noted and physical examination findings are otherwise unremarkable. Biopsy of the mole shows
melanoma, 5.1 mm in depth, without ulceration. The patient undergoes a sentinel lymph node
biopsy, with two of three axillary lymph nodes showing involvement by melanoma. Molecular testing
shows wildtype BRAF.

Which of the following is the most appropriate next step?

 Observation with serial nodal ultrasonogram

 Completion lymph node dissection

 Serial nodal ultrasonogram and adjuvant anti-PD1 therapy

 Completion lymph node dissection and adjuvant immunotherapy

Rationale:
Adjuvant immunotherapy has clearly demonstrated significant recurrence-free survival benefit
versus observation alone for resected stage III melanoma. This is considered the current standard
of care. Multiple studies (MSLT-1, MSLT-2, DeCOG-SLT) have examined the role of completion
lymph node dissection (CLND) in patients with positive sentinel lymph nodes (but were clinically
N0), and none of these studies have shown benefit in melanoma-specific or overall survival. CLND
does offer local control, but this must be balanced against the significant morbidity of surgery, and
thus should not be considered the default option for patients.

References:
Eggermont AMM, Blank CU, Mandala M, et al. Adjuvant pembrolizumab versus placebo in
resected stage III melanoma. N Engl J Med. 2018;378(19):1789-1801.
doi:10.1056/NEJMoa1802357
Faries MB, Thompson JF, Cochran AJ, et al. Completion dissection or observation for
sentinel-node metastasis in melanoma. N Engl J Med. 2017;376(23):2211-2222.
doi:10.1056/NEJMoa1613210

Next
Question 21 of 37

21 A 53-year-old man was diagnosed with metastatic melanoma to the lungs. His melanoma had a
BRAF V600E mutation. He underwent treatment with ipilimumab and nivolumab therapy, with
subsequent disease progression to his liver. His treatment was then switched to BRAF and MEK
inhibitor therapy with dabrafenib and trametinib. Two months into this treatment, his repeat scans
showed decrease in all of his sites of metastases. One week later, he presented with decreased
visual acuity in his left eye. He was seen by an ophthalmologist, with optical coherence tomography
(OCT) demonstrating a serous retinal detachment. He was advised to hold treatment, and his visual
symptoms resolved entirely in one week, with repeat OCT showing improvement of the detachment.

Which of the following should you do now?

 Permanently discontinue dabrafenib and trametinib

 Resume dabrafenib and trametinib, with consideration for a dose reduction

 Stop dabrafenib and continue trametinib

 Discontinue trametinib and add nivolumab

Submit
Submit
Question 21 of 37

21 A 53-year-old man was diagnosed with metastatic melanoma to the lungs. His melanoma had a
BRAF V600E mutation. He underwent treatment with ipilimumab and nivolumab therapy, with
subsequent disease progression to his liver. His treatment was then switched to BRAF and MEK
inhibitor therapy with dabrafenib and trametinib. Two months into this treatment, his repeat scans
showed decrease in all of his sites of metastases. One week later, he presented with decreased
visual acuity in his left eye. He was seen by an ophthalmologist, with optical coherence tomography
(OCT) demonstrating a serous retinal detachment. He was advised to hold treatment, and his visual
symptoms resolved entirely in one week, with repeat OCT showing improvement of the detachment.

Which of the following should you do now?

 Permanently discontinue dabrafenib and trametinib

 Resume dabrafenib and trametinib, with consideration for a dose reduction

 Stop dabrafenib and continue trametinib

 Discontinue trametinib and add nivolumab

Rationale:
Retinal detachments are a known adverse effect of MEK inhibitors, and generally do not cause
irreversible loss of vision or serious eye damage. If retinal detachment improves within three weeks
after therapy interruption, trametinib may be resumed at either the same or lower dose. If it does
not improve within three weeks after therapy interruption, trametinib dose may be reduced or
permanently discontinued. There would be no indication to discontinue dabrafenib, as BRAF
inhibitors are unlikely to cause this symptom. Adding nivolumab to dabrafenib alone would also not
be indicated given the possibility of continuing trametinib, patient‘s treatment response to the
current regimen, and prior disease progression on nivolumab.

References:
Francis JH, Habib LA, Abramson DH, et al. Clinical and morphologic characteristics of MEK
inhibitor-associated retinopathy: differences from central serous chorioretinopathy.
Ophthalmology. 2017;124(12):1788-1798. doi:10.1016/j.ophtha.2017.05.038
Méndez-Martínez S, Calvo P, Ruiz-Moreno O, et al. Ocular adverse events associated with
MEK inhibitors. Retina. 2019;39(8):1435-1450. doi:10.1097/IAE.0000000000002451
Next
Question 22 of 37

22 A 45-year-old man presented with a growing lesion on his right upper arm. Biopsy of the lesion
revealed melanoma with a Breslow depth of 4.2 mm and ulceration. PET-CT scan and brain MRI did
not show any distant metastases. He underwent a wide local excision and lymphoscintigraphy, with
pathology showing complete excision of melanoma. Two sentinel lymph nodes from the right axilla
were removed, both with 1-mm deposit of melanoma. The tumor did not show a BRAF V600
mutation. His Eastern Cooperative Oncology Group performance status is 0.

Which of the following should you recommend now?

 Adjuvant anti-PD1 immunotherapy

 Adjuvant radiation therapy

 Active surveillance

 Completion lymph node dissection

Submit
Submit
Question 22 of 37

22 A 45-year-old man presented with a growing lesion on his right upper arm. Biopsy of the lesion
revealed melanoma with a Breslow depth of 4.2 mm and ulceration. PET-CT scan and brain MRI did
not show any distant metastases. He underwent a wide local excision and lymphoscintigraphy, with
pathology showing complete excision of melanoma. Two sentinel lymph nodes from the right axilla
were removed, both with 1-mm deposit of melanoma. The tumor did not show a BRAF V600
mutation. His Eastern Cooperative Oncology Group performance status is 0.

Which of the following should you recommend now?

 Adjuvant anti-PD1 immunotherapy

 Adjuvant radiation therapy

 Active surveillance

 Completion lymph node dissection

Rationale:
This patient has stage IIIC (T4bN2a) melanoma. Currently, adjuvant nivolumab and pembrolizumab
are FDA-approved treatments for these patients, along with adjuvant dabrafenib and trametinib, for
patients with BRAF-mutant disease. This patient‘s tumor does not have a BRAF mutation. Given
the high risk of relapse with stage IIIC melanoma postsurgery, adjuvant anti-PD1 therapy would be
recommended, and has shown superior relapse-free survival compared to placebo/observation for
these patients. Results from the MSLT-2 and DeCOG-SLT studies demonstrate that completion
lymph node dissection (CLND) is no longer considered the standard recommended treatment for all
patients with positive sentinel lymph node biopsy specimens. Adjuvant radiation would have no role
in sentinel lymph node-positive melanoma.

References:
Eggermont AMM, Blank CU, Mandala M, et al. Adjuvant pembrolizumab versus placebo in
resected stage iii melanoma. N Engl J Med. 2018;378(19):1789-1801.
doi:10.1056/NEJMoa1802357
Ascierto PA, Del Vecchio M, Mandalà M, et al. Adjuvant nivolumab versus ipilimumab in
resected stage IIIB-C and stage IV melanoma (CheckMate 238): 4-year results from a
multicentre, double-blind, randomised, controlled, phase 3 trial [published correction
appears in Lancet Oncol. 2021 Oct;22(10):e428]. Lancet Oncol. 2020;21(11):1465-1477.
doi:10.1016/S1470-2045(20)30494-0
Next
Question 23 of 37

23 A 51-year-old man presented to dermatology after noticing a dark mole on his right arm growing
gradually for the past few months. An excisional biopsy showed at least a 2.5-mm melanoma with a
narrow margin (0.5 mm) of unaffected tissue. The patient has no other suspicious skin lesions and
no palpable adenopathy on the physical examination. He feels well with no other symptoms.
Laboratory test results are unremarkable.

Which of the following is the most appropriate next step?

 Refer for Mohs surgery

 Wide local excision and regular surveillance of the right axilla by physical examination

 Wide local excision and sentinel lymph node biopsy

 Wide local excision and regular surveillance of the right axilla by ultrasonography

Submit
Submit
Question 23 of 37

23 A 51-year-old man presented to dermatology after noticing a dark mole on his right arm growing
gradually for the past few months. An excisional biopsy showed at least a 2.5-mm melanoma with a
narrow margin (0.5 mm) of unaffected tissue. The patient has no other suspicious skin lesions and
no palpable adenopathy on the physical examination. He feels well with no other symptoms.
Laboratory test results are unremarkable.

Which of the following is the most appropriate next step?

 Refer for Mohs surgery

 Wide local excision and regular surveillance of the right axilla by physical examination

 Wide local excision and sentinel lymph node biopsy

 Wide local excision and regular surveillance of the right axilla by ultrasonography

Rationale:
Wide local excision is recommended in melanoma; for patients with disease more than 2 mm in
depth (T3a and above), clinical margins of more than 2 cm are recommended. Biopsy-based
staging of intermediate-thickness or thick primary melanomas provides important prognostic
information and identifies patients with nodal metastases. Mohs surgery is not recommended for
resecting melanoma of this thickness. Patients with intermediate-thickness melanoma without gross
lymph node involvement would first require a sentinel lymph node biopsy and a physical
examination or ultrasonogram for prognostic purposes.

References:
National Comprehensive Cancer Network. NCCN Clinical Practice Guidelines in Oncology:
Melanoma: Cutaneous. Version 2. 2022.
https://www.nccn.org/professionals/physician_gls/pdf/cutaneous_melanoma.pdf
Morton DL, Thompson JF, Cochran AJ, et al. Final trial report of sentinel-node biopsy versus
nodal observation in melanoma. N Engl J Med. 2014;370(7):599-609.
doi:10.1056/NEJMoa1310460

Next
Question 24 of 37

24 A 54-year-old woman presented to the emergency department with a headache. MRI of the brain
showed two brain lesions concerning for metastases but no edema. She had history of stage II
melanoma five years prior, treated with wide local excision and sentinel lymph node biopsy.
Headache improved with pain medications. PET-CT showed hypermetabolic bilateral lung nodules.
Pathology findings from CT-guided needle biopsy demonstrated melanoma with wildtype BRAF and
NRAS mutation. Radiation oncology performed stereotactic radiosurgery (SRS) on the two brain
metastases. The patient is feeling well, with an Eastern Cooperative Oncology Group performance
status of 1. Her headache has resolved and she has no neurologic deficits. Laboratory findings
showed an elevated lactate dehydrogenase (LDH) level, but are otherwise unremarkable.

Which of the following is the most appropriate next step?

 Ipilimumab and nivolumab

 Trametinib

 Dabrafenib and trametinib

 Whole-brain radiation

Submit
Submit
Question 24 of 37

24 A 54-year-old woman presented to the emergency department with a headache. MRI of the brain
showed two brain lesions concerning for metastases but no edema. She had history of stage II
melanoma five years prior, treated with wide local excision and sentinel lymph node biopsy.
Headache improved with pain medications. PET-CT showed hypermetabolic bilateral lung nodules.
Pathology findings from CT-guided needle biopsy demonstrated melanoma with wildtype BRAF and
NRAS mutation. Radiation oncology performed stereotactic radiosurgery (SRS) on the two brain
metastases. The patient is feeling well, with an Eastern Cooperative Oncology Group performance
status of 1. Her headache has resolved and she has no neurologic deficits. Laboratory findings
showed an elevated lactate dehydrogenase (LDH) level, but are otherwise unremarkable.

Which of the following is the most appropriate next step?

 Ipilimumab and nivolumab

 Trametinib

 Dabrafenib and trametinib

 Whole-brain radiation

Rationale:
The patient has stage IV melanoma. Brain metastases were symptomatic and controlled on pain
meds and SRS. Next steps would include systemic therapy with ipilimumab and nivolumab, which
has activity against extracranial as well as intracranial disease. After local treatment of one to three
melanoma brain metastases, adjuvant whole-brain radiation does not provide clinical benefit in
terms of distant intracranial control, survival, or preservation of performance status. The patient
does not have a BRAF V600 mutation, therefore, dabrafenib and trametinib are not recommended.
Trametinib monotherapy is rarely used in the standard management of metastatic melanoma; it has
very modest activity in patients with NRAS mutations and is not recommended in first-line treatment
of metastatic melanoma.

References:
Hong AM, Fogarty GB, Dolven-Jacobsen K, et al. Adjuvant whole-brain radiation therapy
compared with observation after local treatment of melanoma brain metastases: a
multicenter, randomized phase III trial. J Clin Oncol. 2019;37(33):3132-3141.
doi:10.1200/JCO.19.01414
Wolchok JD, Chiarion-Sileni V, Gonzalez R, et al. Overall survival with combined nivolumab
and ipilimumab in advanced melanoma [published correction appears in N Engl J Med. 2018
Nov 29;379(22):2185]. N Engl J Med. 2017;377(14):1345-1356. doi:10.1056/NEJMoa1709684
Tawbi HA, Forsyth PA, Algazi A, et al. Combined Nivolumab and ipilimumab in melanoma
metastatic to the brain. N Engl J Med. 2018;379(8):722-730. doi:10.1056/NEJMoa1805453

Next
Question 25 of 37

25 A 42-year-old woman presents to your clinic after being recently diagnosed with metastatic
melanoma. PET-CT showed lung and liver metastases. She has no relevant medical history. Other
than some fatigue and shortness of breath on exertion, she is asymptomatic. Her Eastern
Cooperative Oncology Group performance status is 1. Laboratory testing reveals a mildly elevated
lactate dehydrogenase (LDH) level of 300 U/L; otherwise, laboratory findings are unremarkable.
MRI of the brain shows no intracranial metastases. Genetic testing demonstrates a BRAF V600E
mutation.

Which of the following is the most appropriate next step?

 Dabrafenib

 Dabrafenib and trametinib

 Ipilimumab and nivolumab

 Ipilimumab

Submit
Submit
Question 25 of 37

25 A 42-year-old woman presents to your clinic after being recently diagnosed with metastatic
melanoma. PET-CT showed lung and liver metastases. She has no relevant medical history. Other
than some fatigue and shortness of breath on exertion, she is asymptomatic. Her Eastern
Cooperative Oncology Group performance status is 1. Laboratory testing reveals a mildly elevated
lactate dehydrogenase (LDH) level of 300 U/L; otherwise, laboratory findings are unremarkable.
MRI of the brain shows no intracranial metastases. Genetic testing demonstrates a BRAF V600E
mutation.

Which of the following is the most appropriate next step?

 Dabrafenib

 Dabrafenib and trametinib

 Ipilimumab and nivolumab

 Ipilimumab

Rationale:
The phase 3 trial CheckMate-067 showed significantly longer overall survival occurred in patients
receiving combination therapy with nivolumab plus ipilimumab than with ipilimumab alone. Another
phase 3 clinical trial (ECOG 6134) compared ipilimumab and nivolumab to dabrafenib and
trametinib in first-line treatment for BRAF V600–mutated advanced melanoma and showed that
ipilimumab and nivolumab was associated with improved overall survival. The DREAMseq phase 3
trial (ECOG 6134) showed that dabrafenib and trametinib had inferior overall survival when
compared with ipilimumab and nivolumab in first-line treatment for advanced BRAF V600–mutated
melanoma. Monotherapy with either dabrafenib or ipilimumab is inferior to combined regimens
dabrafenib and trametinib, and ipilimumab and nivolumab, respectively.

References:
Larkin J, Chiarion-Sileni V, Gonzalez R, et al. Five-year survival with combined nivolumab
and ipilimumab in advanced melanoma. N Engl J Med. 2019;381(16):1535-1546.
doi:10.1056/NEJMoa1910836
Atkins MB, Lee SJ, Chmielowski B, et al. DREAMseq (Doublet, Randomized Evaluation in
Advanced Melanoma Sequencing): a phase III trial—ECOG-ACRIN EA6134. J Clin Oncol.
2021;39(suppl 36):356154-356154. doi:10.1200/JCO.2021.39.36_suppl.356154
Next
Question 26 of 37

26 A 71-year-old man presented to you after being diagnosed with metastatic Merkel cell carcinoma.
He had a growing flesh-colored lesion on his left thigh. PET-CT was done and showed
hypermetabolic liver lesions. Pathology findings from CT-guided liver biopsy showed evidence of
metastatic Merkel cell carcinoma. The patient is feeling well. Eastern Cooperative Oncology Group
performance status of 1. Laboratory test results are unremarkable.

Which of the following should you recommend now?

 Carboplatin and etoposide

 Cisplatin and etoposide

 Ipilimumab and nivolumab

 Pembrolizumab

Submit
Submit
Question 26 of 37

26 A 71-year-old man presented to you after being diagnosed with metastatic Merkel cell carcinoma.
He had a growing flesh-colored lesion on his left thigh. PET-CT was done and showed
hypermetabolic liver lesions. Pathology findings from CT-guided liver biopsy showed evidence of
metastatic Merkel cell carcinoma. The patient is feeling well. Eastern Cooperative Oncology Group
performance status of 1. Laboratory test results are unremarkable.

Which of the following should you recommend now?

 Carboplatin and etoposide

 Cisplatin and etoposide

 Ipilimumab and nivolumab

 Pembrolizumab

Rationale:
Pembrolizumab was studied in a phase 2 trial in patients with advanced Merkel cell carcinoma.
Pembrolizumab demonstrated durable tumor control, a generally manageable safety profile, and
good overall survival compared with historical data from patients treated with first-line
chemotherapy. Chemotherapy with platinum and etoposide was the historical standard of care for
metastatic Merkel cell carcinoma; it was sensitive to chemotherapy regimens used in small cell lung
cancer, but those responses were usually brief. Outcomes observed with anti-PD1 therapy are
better with durable responses. No data are available in comparing ipilimumab and nivolumab to
anti-PD-1 monotherapy; this combination of is associated with a higher rate of immune adverse
events and is therefore not recommend in this setting.

References:
Nghiem P, Bhatia S, Lipson EJ, et al. Durable tumor regression and overall survival in
patients with advanced Merkel cell carcinoma receiving pembrolizumab as first-line therapy.
J Clin Oncol. 2019;37(9):693-702. doi:10.1200/JCO.18.01896
Pectasides D, Moutzourides G, Dimitriadis M, Varthalitis J, Athanassiou A. Chemotherapy
for Merkel cell carcinoma with carboplatin and etoposide. Am J Clin Oncol. 1995;18(5):418-
420. doi:10.1097/00000421-199510000-00011

Next
Question 27 of 37

27 A 56-year-old man presents with an abnormal mole in the left forearm. Biopsy demonstrates a
malignant melanoma that is 1.5 mm in Breslow depth and not ulcerated. The patient is otherwise
asymptomatic. The next step planned in staging and treatment is a wide local excision of the
melanoma.

What other intervention is most appropriate at this time for staging?

 PET imaging

 Cross-sectional imaging

 Sentinel lymph node biopsy

 Wide local excision alone

Submit
Submit
Question 27 of 37

27 A 56-year-old man presents with an abnormal mole in the left forearm. Biopsy demonstrates a
malignant melanoma that is 1.5 mm in Breslow depth and not ulcerated. The patient is otherwise
asymptomatic. The next step planned in staging and treatment is a wide local excision of the
melanoma.

What other intervention is most appropriate at this time for staging?

 PET imaging

 Cross-sectional imaging

 Sentinel lymph node biopsy

 Wide local excision alone

Rationale:
Sentinel lymph node status is an independent predictor of overall survival and is an important
component of staging and prognosis. All patients at risk for regional nodal involvement (at least T1b
disease) should be offered this procedure. PET and cross-sectional imaging are controversial in
early-stage disease and are not an independent predictor of overall survival. Wide local excision
alone is not recommended in patients at risk for regional nodal involvement as it does not provide
accurate staging.

References:
Swetter SM, Tsao H, Bichakjian CK, et al. Guidelines of care for the management of primary
cutaneous melanoma. J Am Acad Dermatol. 2019;80(1):208-250.
doi:10.1016/j.jaad.2018.08.055
Wong SL, Faries MB, Kennedy EB, et al. Sentinel lymph node biopsy and management of
regional lymph nodes in melanoma: American Society of Clinical Oncology and Society of
Surgical Oncology clinical practice guideline update. J Clin Oncol. 2018;36(4):399-413.
doi:10.1200/jco.2017.75.7724
Morton DL, Thompson JF, Cochran AJ, et al. Final trial report of sentinel-node biopsy versus
nodal observation in melanoma. N Engl J Med. 2014;370(7):599-609.
doi:10.1056/nejmoa1310460

Next
Question 28 of 37

28 A 78-year-old man presents with a pigmented lesion on the upper right side of his back that has
been growing in size, as well as getting darker, with irregular margins over the last six months. He
also mentions that he has a lump in his right armpit, which he noticed two months ago, that has
been increasing in size with intermittent bleeding. Biopsy of the lesion and pathology reports
malignant melanoma. He underwent a staging PET/CT, which showed a hyperpigmented lesion on
his upper back along with hypermetabolic axillary lymphadenopathy and multiple liver metastases.
His biopsy sample was sent for next-generation sequencing.

In which of the following genes would you most likely expect to see a mutation?

 BRAF V600K

 NRAS

 BRAF V600E

 KIT

Submit
Submit
Question 28 of 37

28 A 78-year-old man presents with a pigmented lesion on the upper right side of his back that has
been growing in size, as well as getting darker, with irregular margins over the last six months. He
also mentions that he has a lump in his right armpit, which he noticed two months ago, that has
been increasing in size with intermittent bleeding. Biopsy of the lesion and pathology reports
malignant melanoma. He underwent a staging PET/CT, which showed a hyperpigmented lesion on
his upper back along with hypermetabolic axillary lymphadenopathy and multiple liver metastases.
His biopsy sample was sent for next-generation sequencing.

In which of the following genes would you most likely expect to see a mutation?

 BRAF V600K

 NRAS

 BRAF V600E

 KIT

Rationale:
BRAF activating mutations are present in approximately 40% to 60% of patients with metastatic
cutaneous melanomas, among them BRAF V600E is more common compared to BRAF V600K,
accounting for 80% to 90% of cases and 15% of cases, respectively. Other less frequent BRAF
mutations account for less than 5% of cases. NRAS mutations occur in about 15% to 20% of cases.
KIT mutations are more common in patients with chronic sun damage, compared to the above
mutations which are more common in patients without chronic sun damage. Generally, these
mutations are mutually exclusive except in rare cases.

References:
Davies H, Bignell GR, Cox C, et al. Mutations of the BRAF gene in human cancer. Nature.
2002;417(6892):949-954. doi:10.1038/nature00766
Curtin JA, Fridlyand J, Kageshita T, et al. Distinct sets of genetic alterations in melanoma. N
Engl J Med. 2005;353(20):2135-2147. doi:10.1056/NEJMoa050092
Hodis E, Watson IR, Kryukov GV, et al. A landscape of driver mutations in melanoma. Cell.
2012;150(2):251-263. doi:10.1016/j.cell.2012.06.024
Lin YC, Chang YM, Ho JY, et al. C-kit expression of melanocytic neoplasm and association
with clinicopathological parameters and anatomic locations in Chinese people. Am J
Dermatopathol. 2013;35(5):569-575. doi:10.1097/DAD.0b013e318279566a

Next
Question 29 of 37

29 A 23-year-old woman was found to have a stage I melanoma of the left lower back, for which she
had a wide local resection. She has had two prior stage I cutaneous melanomas resected in the
past. Her father has recently been diagnosed with pancreatic cancer. Given her personal and family
history, you consider the possibility of a germline mutation and discuss referral to a genetic
counselor for consultation and testing.

A mutation in which of the following genes is most likely to be identified in this patient?

 CDKN2A

 TP53

 BRCA2

 BAP1

Submit
Submit
Question 29 of 37

29 A 23-year-old woman was found to have a stage I melanoma of the left lower back, for which she
had a wide local resection. She has had two prior stage I cutaneous melanomas resected in the
past. Her father has recently been diagnosed with pancreatic cancer. Given her personal and family
history, you consider the possibility of a germline mutation and discuss referral to a genetic
counselor for consultation and testing.

A mutation in which of the following genes is most likely to be identified in this patient?

 CDKN2A

 TP53

 BRCA2

 BAP1

Rationale:
A mutation in CDKN2A gene (p16) is the most commonly identified mutation in familial forms of
melanoma. It also increases the risk of developing pancreatic cancer. Germline mutations in the
BRCA1-associated protein 1 (BAP1) gene have been identified in families with cutaneous and
uveal melanoma, though would be less common in the described clinical context. TP53 germline
mutations are associated with Li-Fraumeni syndrome, in which there is an increased risk of multiple
cancers; increased risk of melanoma has been reported although not as commonly as with a
CDKN2A mutation. An association between BRCA1/BRCA2 mutations and melanoma has been
noted in some but not all studies.

References:
Begg CB, Orlow I, Hummer AJ, et al. Lifetime risk of melanoma in CDKN2A mutation carriers
in a population-based sample. J Natl Cancer Inst. 2005;97(20):1507-1515.
doi:10.1093/jnci/dji312
Berwick M, Orlow I, Hummer AJ, et al. The prevalence of CDKN2A germ-line mutations and
relative risk for cutaneous malignant melanoma: an international population-based study.
Cancer Epidemiol Biomarkers Prev. 2006;15(8):1520-1525. doi:10.1158/1055-9965.EPI-06-
0270
Njauw CN, Kim I, Piris A, et al. Germline BAP1 inactivation is preferentially associated with
metastatic ocular melanoma and cutaneous-ocular melanoma families. PLoS One.
2012;7(4):e35295. doi:10.1371/journal.pone.0035295
Next
Question 30 of 37

30 A 56-year-old woman presented to the oncology clinic with a recently diagnosed a 1-mm deep,
ulcerated melanoma in the right thigh. She underwent primary resection and sentinel lymph node
resection revealing one (of one) positive lymph nodes with 1 mm of tumor involvement in the node.
PET/CT imaging showed no evidence of distant disease

Which of the following should you do now?

 Surveillance with nodal ultrasonogram every three months

 Complete lymph node dissection

 Physical examination every three months

Submit
Submit
Question 30 of 37

30 A 56-year-old woman presented to the oncology clinic with a recently diagnosed a 1-mm deep,
ulcerated melanoma in the right thigh. She underwent primary resection and sentinel lymph node
resection revealing one (of one) positive lymph nodes with 1 mm of tumor involvement in the node.
PET/CT imaging showed no evidence of distant disease

Which of the following should you do now?

 Surveillance with nodal ultrasonogram every three months

 Complete lymph node dissection

 Physical examination every three months

Rationale:
According to the MSLT II trial, patients who have localized melanoma with clinically negative nodal
disease and show a positive sentinel lymph node can forego full nodal dissection with adequate
follow-up, consisting of observation and nodal ultrasonogram every three months. The omission of
full nodal dissection in patients with clinical surveillance showed no negative impact no melanoma-
specific survival (86% +/- 1.3% and 86% +/- 1.2%, respectively) and lowered the risk for
lymphedema from 21.4% to 6%. Once-in-a-lifetime nodal ultrasonogram is not a recommended
strategy. To date, nodal dissection has not consistently proven any impact on overall survival in
patients with melanoma.

References:
Faries MB, Thompson JF, Cochran AJ, et al. Completion Dissection or Observation for
Sentinel-Node Metastasis in Melanoma. N Engl J Med. 2017;376(23):2211-2222.
doi:10.1056/NEJMoa1613210
Leiter U, Stadler R, Mauch C, et al. Final Analysis of DeCOG-SLT Trial: No Survival Benefit
for Complete Lymph Node Dissection in Patients With Melanoma With Positive Sentinel
Node. J Clin Oncol. 2019;37(32):3000-3008. doi:10.1200/JCO.18.02306

Next
Question 31 of 37

31 A 52-year-old man presents with a six-month history of weight loss and abdominal pain. The patient
has a history of stage II oral mucosal melanoma diagnosed three years ago, treated with wide local
excision and sentinel lymph node biopsy. Cross-sectional imaging reveals multiple liver lesions.
Biopsy of a liver lesion reveals melanoma histologically similar to his original mucosal melanoma.

Which of the following mutations is most likely to be found in this patient‘s tumor?

 BRAF

 KIT

 NRAS

 NF1

Submit
Submit
Question 31 of 37

31 A 52-year-old man presents with a six-month history of weight loss and abdominal pain. The patient
has a history of stage II oral mucosal melanoma diagnosed three years ago, treated with wide local
excision and sentinel lymph node biopsy. Cross-sectional imaging reveals multiple liver lesions.
Biopsy of a liver lesion reveals melanoma histologically similar to his original mucosal melanoma.

Which of the following mutations is most likely to be found in this patient‘s tumor?

 BRAF

 KIT

 NRAS

 NF1

Rationale:
KIT mutations are more common in mucosal melanoma (15% of cases) compared to cutaneous
melanoma, whereas BRAF and NRAS mutations are less common. NF1 mutations are more
common in sun-exposed skin. Patients with metastatic mucosal melanoma have been treated
successfully at times with immunotherapy, but in general response rates are lower than in patients
with cutaneous melanoma. KIT targeted therapy has also shown some benefit, but responses tend
to be transient.

References:
Gutiérrez-Castañeda LD, Nova JA, Tovar-Parra JD. Frequency of mutations in BRAF, NRAS,
and KIT in different populations and histological subtypes of melanoma: a systemic review.
Melanoma Res. 2020;30(1):62-70. doi:10.1097/CMR.0000000000000628
Tyrrell H, Payne M. Combatting mucosal melanoma: recent advances and future
perspectives. Melanoma Manag. 2018;5(3):MMT11. Published 2018 Oct 8. doi:10.2217/mmt-
2018-0003

Next
Question 32 of 37

32 A 46-year-old man presented to the emergency department with a new-onset fever (102.2 F [39 C])
and chills, without any other localizing symptoms. One month prior, he was diagnosed with a BRAF
V600E–mutated metastatic melanoma to the lung, liver, and bone. He started treatment with
dabrafenib plus trametinib. An extensive infectious workup revealed no abnormalities.

Which of the following is the most likely etiology of this patient‘s fever?

 Neoplastic fever from melanoma

 Pyrexia from BRAF/MEK inhibitors

 Pneumocystis jirovecii pneumonia (PCP)

Submit
Submit
Question 32 of 37

32 A 46-year-old man presented to the emergency department with a new-onset fever (102.2 F [39 C])
and chills, without any other localizing symptoms. One month prior, he was diagnosed with a BRAF
V600E–mutated metastatic melanoma to the lung, liver, and bone. He started treatment with
dabrafenib plus trametinib. An extensive infectious workup revealed no abnormalities.

Which of the following is the most likely etiology of this patient‘s fever?

 Neoplastic fever from melanoma

 Pyrexia from BRAF/MEK inhibitors

 Pneumocystis jirovecii pneumonia (PCP)

Rationale:
Pyrexia defined as a temperature of 101.3 F (38.5 C) or greater is the most common adverse event
associated with dabrafenib plus trametinib in both the adjuvant and metastatic setting, with an
incidence of around 55%. Most of these cases are grade 1 or 2 events and are the most frequent
adverse event leading to treatment discontinuation. Pyrexia typically occurs early in treatment, with
incidence decreasing over time. Holding treatment with dabrafenib and trametinib at the onset of
pyrexia will often interrupt the episode and full-dose BRAF/MEK inhibition can be resumed. The
diagnosis of neoplastic fever is extremely uncommon in melanoma; organic causes such as
infections and medications should be ruled out. To date, BRAF/MEK inhibitor therapy has not been
associated with a higher risk of PCP infection.

References:
Schadendorf D, Robert C, Dummer R, et al. Pyrexia in patients treated with dabrafenib plus
trametinib across clinical trials in BRAF-mutant cancers. Eur J Cancer. 2021;153:234-241.
doi:10.1016/j.ejca.2021.05.005
Robert C, Grob JJ, Stroyakovskiy D, et al. Five-Year Outcomes with Dabrafenib plus
Trametinib in Metastatic Melanoma. N Engl J Med. 2019;381(7):626-636.
doi:10.1056/NEJMoa1904059

Next
Question 33 of 37

33 A 59-year-old man with a history of hypertension reports new-onset itching of a pigmented mole
that he had noticed on his left upper back for about five years. The patient had also seen the
mole increase in size and change in color. He was evaluated by his dermatologist, who did a
biopsy of this lesion for concerns of cutaneous melanoma. Biopsy results confirmed the lesion to be
malignant melanoma of 2.9 mm depth, Clark Level IV, nodular type, three mitosis/ mm^2, ulceration
absent, brisk tumor-infiltrating lymphocytes, regression, and lymphovascular invasion present.
There were no microsatellites, perineural invasion, and in-transit metastases. The patient underwent
complete excision of the tumor with negative margins, and a lymphoscintigraphy procedure resulted
in three sentinel lymph nodes. One of the three lymph nodes showed a focus of melanoma
morphologically and by immunohistochemistry that measured 2.1 mm. Blood test results, including
lactate dehydrogenase (LDH), are normal, and a whole-body PET scan and brain MRI showed no
evidence of metastases. The tumor is assigned AJCC Stage IIIA (T2a, N1a, M0). The tumor testing
for BRAF mutation showed wild-type BRAF. The patient is referred for consultations to discuss risk
stratification and postoperative treatment recommendations to improve long-term outcomes.

Which of the following is the most appropriate next step?

 Anti PD-1 antibody treatment for one year

 Anti CTLA-4 antibody treatment for one year

 Anti BRAF plus anti MEK treatment for one year

 High-dose interferon alfa treatment for one year

Submit
Submit
Question 33 of 37

33 A 59-year-old man with a history of hypertension reports new-onset itching of a pigmented mole
that he had noticed on his left upper back for about five years. The patient had also seen the
mole increase in size and change in color. He was evaluated by his dermatologist, who did a
biopsy of this lesion for concerns of cutaneous melanoma. Biopsy results confirmed the lesion to be
malignant melanoma of 2.9 mm depth, Clark Level IV, nodular type, three mitosis/ mm^2, ulceration
absent, brisk tumor-infiltrating lymphocytes, regression, and lymphovascular invasion present.
There were no microsatellites, perineural invasion, and in-transit metastases. The patient underwent
complete excision of the tumor with negative margins, and a lymphoscintigraphy procedure resulted
in three sentinel lymph nodes. One of the three lymph nodes showed a focus of melanoma
morphologically and by immunohistochemistry that measured 2.1 mm. Blood test results, including
lactate dehydrogenase (LDH), are normal, and a whole-body PET scan and brain MRI showed no
evidence of metastases. The tumor is assigned AJCC Stage IIIA (T2a, N1a, M0). The tumor testing
for BRAF mutation showed wild-type BRAF. The patient is referred for consultations to discuss risk
stratification and postoperative treatment recommendations to improve long-term outcomes.

Which of the following is the most appropriate next step?

 Anti PD-1 antibody treatment for one year

 Anti CTLA-4 antibody treatment for one year

 Anti BRAF plus anti MEK treatment for one year

 High-dose interferon alfa treatment for one year

Rationale:
Melanoma metastases to regional lymph nodes represent a critical prognostic factor in intermediate
thickness cutaneous melanoma and confer reduced melanoma-specific survival. The FDA
approved High-dose interferon alfa for one year due to its modest recurrence-free survival and
marginal overall survival benefit. The newer immune checkpoint inhibitor agents have shownÂ
unequivocal recurrence-free survival and overall survival benefit in patients with stage III melanoma
when administered for one year. Anti-PD-1 treatment has been determined to be superior and safer
than anti-CTLA-4 treatment and is a preferred first choice. The patient‘s lymph node disease
burden also makes this the most appropriate treatment option. In patients with BRAF-mutated
tumors, anti-BRAF plus anti-MEK treatment for one year is FDA approved since it improves event-
free survival and overall survival. Our patient does not harbor BRAF mutated melanoma; anti-BRAF
plus anti-MEK treatment is not recommended. Although an approved option, high-dose interferon
alfa is not favored due to its marginal benefit and significant side effects. ThereforeÂ, the correct
answer is A.

References:
Long GV, Hauschild A, Santinami M, et al. Adjuvant Dabrafenib plus Trametinib in Stage III
BRAF-Mutated Melanoma. N Engl J Med. 2017;377(19):1813-1823. PMID: 28891408.

Next
Question 34 of 37

34 A 70-year-old man presents with slowly progressive dyspnea, back pain, and mental status change
to the emergency room. He has a history of malignant melanoma on the right cheek from 3 years
ago, which was resected with negative margins and was determined to be stage I. Sentinel lymph
nodes were not assessed at that time. In the emergency department, a brain MRI with and without
intravenous gadolinium revealed a 2-cm lesion in the left parietal lobe and a 1-cm lesion in the right
cerebellar hemisphere. Both were associated with vasogenic edema. A CT scan with intravenous
contrast of the chest, abdomen, and pelvis showed extensive liver and bone metastases and one
pulmonary mass measuring 2.5 cm in diameter at the posterior aspect of the right lower lobe. The
patient was given one dose of dexamethasone 10 mg IV. A CT-guided lung biopsy of the right lower
lobe mass was performed upon admission to the hospital and confirmed melanoma.

Which diagnostic test should be requested before selecting first-line systemic therapy?

 FGFR3 gene mutation by PCR

 BRAF gene mutation V600 by PCR

 EGFR gene mutation exon 20 by PCR

 KRAS gene mutation by PCR

Submit
Submit
Question 34 of 37

34 A 70-year-old man presents with slowly progressive dyspnea, back pain, and mental status change
to the emergency room. He has a history of malignant melanoma on the right cheek from 3 years
ago, which was resected with negative margins and was determined to be stage I. Sentinel lymph
nodes were not assessed at that time. In the emergency department, a brain MRI with and without
intravenous gadolinium revealed a 2-cm lesion in the left parietal lobe and a 1-cm lesion in the right
cerebellar hemisphere. Both were associated with vasogenic edema. A CT scan with intravenous
contrast of the chest, abdomen, and pelvis showed extensive liver and bone metastases and one
pulmonary mass measuring 2.5 cm in diameter at the posterior aspect of the right lower lobe. The
patient was given one dose of dexamethasone 10 mg IV. A CT-guided lung biopsy of the right lower
lobe mass was performed upon admission to the hospital and confirmed melanoma.

Which diagnostic test should be requested before selecting first-line systemic therapy?

 FGFR3 gene mutation by PCR

 BRAF gene mutation V600 by PCR

 EGFR gene mutation exon 20 by PCR

 KRAS gene mutation by PCR

Rationale:
Metastatic melanoma with a BRAF gene mutation V600 by PCR indicates treatment with dual
BRAF/MEK inhibition.

References:
Ascierto PA, Dummer R, Gogas HJ, et al. Update on tolerability and overall survival in
COLUMBUS: landmark analysis of a randomised phase 3 trial of encorafenib plus
binimetinib vs vemurafenib or encorafenib in patients with BRAF V600-mutant melanoma.
Eur J Cancer. 2020;126:33-44. PMID: 31901705.

Next
Question 35 of 37

35 A 31-year-old woman presents with a 2.1 mm deep, ulcerated mitoses 2 per high power field (HPF)
melanoma on the left arm. She undergoes a wide local excision with 2Â cm margins and sentinel
lymph node biopsy of the left axilla. She has 1 of 2 positive sentinel nodes. The positive sentinel
lymph node demonstrates a 0.6 mm focus of cells with no extracapsular extension. Staging PET/CT
is negative for metastatic disease, and the tumor is BRAF wild-type.

Which of the following is the most appropriate next step?

 Completion axillary lymph node dissection with adjuvant combined PD1/anti-CTLA4 therapy.

 Ultrasound surveillance of the lymph node basin every 4 months and offer adjuvant PD1 therapy.

 Completion axillary lymph node dissection with adjuvant radiation therapy.

 Follow up with history, physical examination, and labatory studies.

Submit
Submit
Question 35 of 37

35 A 31-year-old woman presents with a 2.1 mm deep, ulcerated mitoses 2 per high power field (HPF)
melanoma on the left arm. She undergoes a wide local excision with 2Â cm margins and sentinel
lymph node biopsy of the left axilla. She has 1 of 2 positive sentinel nodes. The positive sentinel
lymph node demonstrates a 0.6 mm focus of cells with no extracapsular extension. Staging PET/CT
is negative for metastatic disease, and the tumor is BRAF wild-type.

Which of the following is the most appropriate next step?

 Completion axillary lymph node dissection with adjuvant combined PD1/anti-CTLA4 therapy.

 Ultrasound surveillance of the lymph node basin every 4 months and offer adjuvant PD1 therapy.

 Completion axillary lymph node dissection with adjuvant radiation therapy.

 Follow up with history, physical examination, and labatory studies.

Rationale:
This is a patient who presents with stage 3C, T3bN1aM0 melanoma. Two significant decisions
need to be made in this patient regarding the management of the lymph node basin and the role of
adjuvant therapy. Basin options would include ultrasound surveillance every four months or
completion of lymph node dissection regarding the lymph node. The risks and benefits of this
approach should be discussed with the patient. The DeCOG-SLT Trial and MSLT 2 (Completion
Dissection or Observation for Sentinel-Node Metastasis in Melanoma) found no change in overall
survival with dissection versus observation. However, with observation, patients are more likely to
have lymph node basin recurrence. Numerous studies, including "Adjuvant Nivolumab versus
Ipilimumab in resected stage III or IV Melanoma," have demonstrated a significantly longer
recurrence-free survival with adjuvant therapy. The patient is not a candidate for adjuvant
BRAF/MEKi as the tumor was BRAF wild type.

References:
Leiter U, Stadler R, Mauch C, et al. No Survival Benefit for Complete Lymph Node Dissection
in Patients With Melanoma With Positive Sentinel Node. J Clin Oncol. 2019 Nov
10;37(32):3000-3008. PMID: 31557067.
Weber J, Mandala M, Del Vecchio M, et al. Adjuvant Nivolumab versus Ipilimumab in
Resected Stage III or IV Melanoma. N Engl J Med. 2017 Nov 9;377(19):1824-1835. PMID:
28891423.
Faries MB, Thompson JF, Cochran AJ, et al. Completion Dissection or Observation for
Sentinel-Node Metastasis in Melanoma. N Engl J Med. 2017 Jun 8;376(23):2211-2222. PMID:
28591523.

Next
Question 36 of 37

36 A 70-year-old woman with no significant past medical history presents a three month history of a
pink, firm skin nodule on the right lower leg, which has been slowly progressing. There is a palpable
lymph node in the right inguinal region, firm, fixed, and non-tender measuring approximately 2 cm in
diameter. She has had no B symptoms. Punch biopsy of the skin lesion reveals CK20 positive, TTF-
1 negative, infiltrative, malignant appearing cells, with brisk tumor-infiltrating lymphocytes. PET/CT
scan revealed a 2 cm right inguinal lymph node with FDG uptake and maximum SUV of 10, as well
as multiple FDG avid liver and lung lesions ranging between 1 and 3 cm in diameter.

Which of the following is the most appropriate next step?

 Cisplatin

 Cemiplimab-rwlc

 Pembrolizumab

 Topotecan

Submit
Submit
Question 36 of 37

36 A 70-year-old woman with no significant past medical history presents a three month history of a
pink, firm skin nodule on the right lower leg, which has been slowly progressing. There is a palpable
lymph node in the right inguinal region, firm, fixed, and non-tender measuring approximately 2 cm in
diameter. She has had no B symptoms. Punch biopsy of the skin lesion reveals CK20 positive, TTF-
1 negative, infiltrative, malignant appearing cells, with brisk tumor-infiltrating lymphocytes. PET/CT
scan revealed a 2 cm right inguinal lymph node with FDG uptake and maximum SUV of 10, as well
as multiple FDG avid liver and lung lesions ranging between 1 and 3 cm in diameter.

Which of the following is the most appropriate next step?

 Cisplatin

 Cemiplimab-rwlc

 Pembrolizumab

 Topotecan

Rationale:
Immunotherapy with pembrolizumab was approved in 2019 by the FDA for the treatment of
advanced Merkel cell skin cancer based on an overall response rate of 56% (complete response of
24% and partial response of 32%) in clinical trial Keynote-017 which was nonrandomized, open-
label. Avelumab was approved in 2017 as well. Chemotherapy can lead to high response rates
but has minimal impact on survival due to the short progression-free survival associated with
chemotherapy. Cemiplimab-rwlc is approved for advanced squamous cell skin cancer.

References:
Nghiem P, Bhatia S, Lipson EJ, et al. Durable Tumor Regression and Overall Survival in
Patients With Advanced Merkel Cell Carcinoma Receiving Pembrolizumab as First-Line
Therapy. J Clin Oncol. 2019;37(9):693-702. PMID: 30726175.

Next
Question 37 of 37

37 A 60-year-old man presents to the oncology clinic to start treatment for metastatic merkel cell
carcinoma after his disease had progressed on chemotherapy. He had received six cycles of
carboplatin and etoposide with the initial partial response after the first three cycles and progression
after his sixth cycle. He will start 800 mg avelumab to be infused intravenously over 60 minutes. His
HIV and hepatitis serologies are negative.

Which of the following is the most appropriate next step?

 One liter of normal saline

 Oral ondansetron

 Acetaminophen and antihistamine

 Dexamethasone

Submit
Submit
Question 37 of 37

37 A 60-year-old man presents to the oncology clinic to start treatment for metastatic merkel cell
carcinoma after his disease had progressed on chemotherapy. He had received six cycles of
carboplatin and etoposide with the initial partial response after the first three cycles and progression
after his sixth cycle. He will start 800 mg avelumab to be infused intravenously over 60 minutes. His
HIV and hepatitis serologies are negative.

Which of the following is the most appropriate next step?

 One liter of normal saline

 Oral ondansetron

 Acetaminophen and antihistamine

 Dexamethasone

Rationale:
One-quarter of patients who receive avelumab exhibit a severe infusion reaction; hence, the need
for premedication. Avelumab is the only immune checkpoint inhibitor that requires premedication.
Pre-chemotherapy hydration is not needed for kidney protection and ondansetron is not needed for
nausea and vomiting prevention. Dexamethasone should not be used because of its effect on the
immune response that avelumab is trying to elicit.

References:
D‘Angelo SP, Bhatia S, Brohl AS, Hamid O, Mehnert JM, Terheyden P, Shih KC, Brownell I,
Lebb C, Lewis KD, Linette GP, Milella M, Georges S, Shah P, Ellers-Lenz B, Bajars M, Gzel G,
Nghiem PT. Avelumab in patients with previously treated metastatic Merkel cell carcinoma:
long-term data and biomarker analyses from the single-arm phase 2 JAVELIN Merkel 200
trial. J Immunother Cancer. 2020 May;8(1):e000674. doi: 10.1136/jitc-2020-000674. PMID:
32414862; PMCID: PMC7239697.
Avelumab: HIGHLIGHTS OF PRESCRIBING INFORMATION

Continue
Back to ASCO Question Bank

Assessment Results

Assessment Name
2022 Melanoma and Skin Cancers

Created
07/12/2022
Completed Date
07/12/2022

Topics

Melanoma and Skin Cancers

Retake Assessment

Assessment Score
27.03%
Correct

72.97%
Incorrect

10 of 37 questions correct

Assessment Review

Question 1:
A 69-year-old man with a history of hypertension,
diabetes, and HIV presents with a rapidly enlarging lesion
over his left shoulder. He reports a significant history of
sun exposure. He has a 2 cm non-tender, raised, lightly
pigmented lesion over the left shoulder on examination.
Left axillary lymphadenopathy is palpated. Biopsy of the
skin lesion shows a dermal mass extending into the
subcutis, composed of strands of uniform, round, blue
cells, positive for CK20 perinuclear staining, CD56,
chromogranin, and synaptophysin, and negative for S100,
CK7, and thyroid-transcription factor-1.
Fluorodeoxyglucose (FDG)-PET/CT shows
hypermetabolic enlarged left axillary nodes and numerous
pulmonary and liver lesions. A percutaneous biopsy of a
liver lesion shows similar findings to the skin lesion biopsy.
Which of the following is the recommended first-line
treatment?

Your Answer
Carboplatin, etoposide, and atezolizumab
Correct Answer
Avelumab

Rationale

This is a patient with metastatic Merkel cell carcinoma


(MCC). He has several risk factors, including older age
(>50), immunosuppression (HIV), and a history of
significant sun exposure. The described pathologic
findings are typical for this tumor type, including
features of both epithelial and neuroendocrine cells,
particularly CK20 positivity. Negativity for TTF-1 and
S100 help exclude small cell carcinoma of the lung and
melanoma, respectively. Avelumab is a monoclonal
anti-PDL1 antibody with demonstrated activity in
advanced MCC and is currently approved in the first-
line setting. Ipilimumab and the combination of
ipilimumab and nivolumab are not approved in the first-
line setting for advanced MCC at this time.
Carboplatin, etoposide, and atezolizumab are currently
approved first-line regimens for the lung's extensive-
stage small cell carcinoma. It is not approved at this
time in the first-line setting for advanced MCC.

References

Kaufman HL, Russell J, Hamid O, et al. Avelumab in


patients with chemotherapy-refractory metastatic Merkel
cell carcinoma: a multicentre, single-group, open-label,
phase 2 trial. Lancet Oncol. 2016;17(10):1374-1385.
https://pubmed.ncbi.nlm.nih.gov/27592805/

Incorrect
Question 2:
A 69-year-old man with oculocutaneous albinism (OCA)
and a history of numerous cutaneous squamous cell
carcinomas presents with an enlarging right neck mass.
He has a new skin lesion over the right lower face and a
large nodal mass on the right side of his neck—biopsy of
the skin lesion and fine-needle aspiration of the right
cervical nodal mass show squamous cell carcinoma. No
distant metastases are identified. The skin lesion is
resected, and he receives radiation therapy to the right
neck concurrently with chemotherapy. After an initial
partial response, he has progressive disease in the right
neck. Fluorodeoxyglucose (FDG)-PET/CT shows
increased hypermetabolic right cervical adenopathy and
numerous new pulmonary lesions. CT-guided biopsy of a
pulmonary lesion shows squamous cell carcinoma. 
Which of the following would be the best management at
this time?

Your Answer
Vismodegib

Correct Answer
Cemiplimab

Rationale

This is a patient with now metastatic cutaneous


squamous cell carcinoma (SCC). Cemiplimab is an
anti-PD1 antibody that is effective in this setting.
Vismodegib is an SMO inhibitor that blocks the
hedgehog pathway's activation and has demonstrated
efficacy in basal cell carcinoma. Systemic
chemotherapy has limited efficacy in advanced
cutaneous SCC. EGFR inhibitors such as gefitinib and
erlotinib have also been shown to have limited efficacy
in advanced cutaneous SCC.
References

Migden MR, Rischin D, Schmults CD, et al. PD-1


Blockade with Cemiplimab in Advanced Cutaneous
Squamous-Cell Carcinoma. N Engl J Med.
2018;379(4):341-351.
https://pubmed.ncbi.nlm.nih.gov/29863979/

Incorrect

Question 3:
A 48-year-old man previously in good health presents with
an enlarging mole over his right foot's dorsum. A biopsy
shows melanoma, 1.1 mm in thickness, with ulceration, 2
mitoses per mm^2, with a positive posterior margin. Wide
local excision shows no residual melanoma. Sentinel
inguinal lymph node biopsy is negative. On a follow-up
visit 6 months later, a 0.5 cm subcutaneous nodule is
noted over the right shin. No other skin lesions are noted,
and there is no palpable inguinal or other
lymphadenopathy on the examination. The biopsy of the
subcutaneous nodule shows melanoma. No BRAF
mutation is identified. FDG-PET/CT from top of the skull to
toes shows a hypermetabolic subcutaneous nodule over
the right shin, with no other evidence of metastatic
disease. MRI of the brain is negative for intracranial
metastasis. The patient's ECOG performance status is 0.
Hematologic and chemistry profiles are within normal
limits. 
Which of the following is the most appropriate
management strategy?

Correct Answer
Complete surgical excision of the subcutaneous
nodule, followed by 1 year of nivolumab

Rationale

This patient with initially stage IIA melanoma (T2b N0)


of the right foot, currently with evidence of a single in-
transit metastasis, which now represents stage IIIB
disease. As there is no other evidence of metastatic
disease, and the isolated in-transit metastasis appears
to be resectable, the recommended primary treatment
in this setting is complete surgical excision of the in-
transit to clear margins. Following excision,
recommended options include observation and
adjuvant therapy. It would be reasonable to offer
adjuvant immunotherapy for one year in this case.
Complete lymph node dissection is not indicated in the
absence of clinical nodal involvement.

References

Read RL, Thompson JF. Managing in-transit melanoma


metastases in the new era of effective systemic therapies
for melanoma. Expert Rev Clin Pharmacol.
2019;12(12):1107-1119.
https://pubmed.ncbi.nlm.nih.gov/31687857/

Correct

Question 4:
A 26-year-old woman presents with an enlarging mole on
her right upper back. Biopsy shows melanoma, 0.5 mm
thick, without ulceration, 1 mitosis per mm^2, with close
margins. No palpable lymphadenopathy is noted on exam.
She undergoes a wide local excision, which shows no
residual melanoma. She has a history of a previous early-
stage melanoma excised two years ago from her left arm.
She reports a history of melanoma in her mother and two
of her brothers, a history of breast cancer in an aunt (non-
blood relative), and pancreatic cancer in a maternal uncle.
Which of the following is the most commonly identified
mutated gene in familial forms of melanoma?

Your Answer
BAP1

Correct Answer
CDKN2A

Rationale

CDKN2A is the most commonly identified mutated


gene in familial forms of melanoma. This gene
encodes two proteins, p16, and p14ARF; p16 is the
more commonly impaired protein. The prevalence of
these mutations in families with hereditary melanoma
is highly variable and ranges from 20% to 60% in
certain families, depending on geographic location.
The prevalence of CDKN2A germline mutations in the
general population is much lower, estimated at around
1.2% in individuals with a single primary melanoma
and 2.9% in individuals with multiple primary
melanomas. A weaker association between BRCA1
and BRCA2 mutations and melanoma has been found
in some but not all studies. BAP1 germline mutations
have been identified in families with cutaneous and
uveal melanoma.

References

Goldstein AM, Chan M, Harland M, et al. High-risk


melanoma susceptibility genes and pancreatic cancer,
neural system tumors, and uveal melanoma across
GenoMEL. Cancer Res. 2006;66(20):9818-9828.
https://pubmed.ncbi.nlm.nih.gov/17047042/

Incorrect

Question 5:
A 43-year-old woman presents to her dermatologist with a
pigmented lesion on the abdomen. Physical examination
reveals an ulcerated neoplasm with concerning features
on dermoscopy. A shave biopsy reveals a 5.70 mm
Breslow, Clarks level V, ulcerated, nodular melanoma with
5 mitoses/mm^2, moderate tumor-infiltrating lymphocytes
(TIL) no evidence of satellitosis or regression. There is the
presence of Angio-lymphatic, and perineural invasion and
margins are involved. A  wide local excision (WLE)  and
sentinel lymph node biopsy are performed and tested. Two
lymph nodes are positive for melanoma, and the largest
metastatic deposit is 4 mm. 
Which of the following mutations is most likely to be found
on genetic analysis of the primary specimen?

Your Answer
GNA11 c.547C > T (R183C) mutation

Correct Answer
BRAF c.1799T > A (V600E) mutation

Rationale

BRAF exon 15 (37 to 50%), NRAS codon 61 (13


to 25%), and NF1 (12%) mutations account for the
majority of mutations found in cutaneous melanoma.
Based on its association with chronically sun-damaged
(CSD) skin, cutaneous melanoma can be categorized
as CSD or non-CSD melanoma. CSD and non-CSD
melanoma have different clinicopathological
characteristics and are associated with different driver
mutations. CSD melanomas typically arise in older
(> 55 years) patients on sun-exposed areas
(head/neck, dorsal surfaces of distal extremities). They
are associated with specific driver mutations
(BRAF non-V600E, NRAS, NF1, or KIT) and genetic
signatures of UV-induced DNA damage [G > T (UVA)
or C > T (UVB)] transitions. In contrast, non-CSD
melanomas typically arise in younger (< 55 years)
patients on intermittently sun-exposed areas (trunk,
proximal extremities), and are associated with BRAF
V600E/K driver mutations, and often lack the genetic
signature of UV mutagenesis. PIK3CA exon 9
mutations are observed in up to 26% of breast cancer.
Although PTEN loss is seen in 7 to 28% of melanoma,
these mutations typically occur in exon 5 rather than
exon 9.

References

Curtin JA, Fridlyand J, Kageshita T, et al. Distinct sets of


genetic alterations in melanoma. N Engl J Med.
2005;353(20):2135-2147.
https://pubmed.ncbi.nlm.nih.gov/16291983/

Hodis E, Watson IR, Kryukov GV, et al. A landscape of


driver mutations in melanoma. Cell. 2012;150(2):251-263.
https://pubmed.ncbi.nlm.nih.gov/22817889/

Cancer Genome Atlas Network. Genomic Classification of


Cutaneous Melanoma. Cell. 2015;161(7):1681-1696.
https://pubmed.ncbi.nlm.nih.gov/26091043/

Incorrect
Question 6:
A 61-year-old man presents with floaters in his right eye,
which appeared three weeks ago. On fundoscopic and
ultrasonographic examination, he is found to have a right
choroidal pigmented lesion. Upon further evaluation, it is
determined to be a uveal melanoma.
Which of the following is the most commonly identified
somatic mutation in this tumor type?

Correct Answer
GNAQ

Rationale

GNAQ and GNA11 are the most commonly identified


somatic mutations in uveal melanoma, with a mutation
in either gene occurring in about 80% of cases. Other
commonly identified mutations in this tumor type
include BAP1, SF3B1, and EIF1AX. Unlike in
cutaneous melanoma, mutations in BRAF, NRAS, and
KIT are rarely found in uveal melanoma.

References

Van Raamsdonk CD, Bezrookove V, Green G, et al.


Frequent somatic mutations of GNAQ in uveal melanoma
and blue naevi. Nature. 2009;457(7229):599-602.
https://pubmed.ncbi.nlm.nih.gov/19078957/

Correct
Question 7:
A 45-year-old man presents with floaters in his left eye,
which appeared 4 days ago. On fundoscopic and
ultrasonographic evaluation, he is found to have a left
ciliary body pigmented lesion. Upon further evaluation, it is
determined to be a primary ocular melanoma. He reports a
family history of cutaneous melanoma in two of his
younger sisters, mesothelioma in his mother, and renal
cell carcinoma in a maternal uncle. 
A germline mutation in which of the following genes is
most likely to be found in this patient?

Your Answer
BRAF

Correct Answer
BAP1

Rationale

Germline mutations in the BRCA1-associated gene 1


(BAP1) have been identified in families with cutaneous
and uveal melanoma and linked to a BAP-1
tumor predisposition syndrome. The risk of developing
mesothelioma and renal cell carcinoma also appears
to be increased. GNA11 and GNAQ somatic mutations
are commonly identified in uveal melanoma; germline
mutations in these genes are not commonly
identified. Somatic mutations in BRAF are commonly
present in cutaneous melanoma. 

References

Njauw CN, Kim I, Piris A, et al. Germline BAP1


inactivation is preferentially associated with metastatic
ocular melanoma and cutaneous-ocular melanoma
families. PLoS One. 2012;7(4):e35295.
https://pubmed.ncbi.nlm.nih.gov/22545102/
Testa JR, Cheung M, Pei J, et al. Germline BAP1
mutations predispose to malignant mesothelioma. Nat
Genet. 2011;43(10):1022-1025.
https://pubmed.ncbi.nlm.nih.gov/21874000/

Incorrect

Question 8:
A 36-year-old woman presents with an enlarging
pigmented lesion on her right arm. Biopsy shows a nodular
melanoma, 2.5 mm in depth, 3 mitoses per mm^2, with
ulceration and positive margins. Wide local excision and
axillary sentinel lymph node biopsy are performed. There
is no residual disease on the wide local excision sample.
The sentinel node shows a 1.5 mm focus of metastatic
melanoma. Molecular testing shows no BRAF mutation.
Tumor PD-L1 expression is found to be 1%. PET/CT and
MRI of the brain are negative for distant metastatic
disease.
Which of the following is the best next step in
management?

Your Answer
Lymph node dissection

Correct Answer
Nivolumab

Rationale

This is a patient with stage IIIB melanoma. Immediate


completion lymph node dissection is no longer
recommended following a positive sentinel lymph node
biopsy; periodic monitoring of the nodal basin is
performed instead to evaluate for recurrence. Adjuvant
therapy would generally be recommended, or at least
considered, for patients with stage IIIB melanoma.
Nivolumab has been shown to significantly improve
recurrence-free survival in the adjuvant setting
compared to ipilimumab regardless of PD-L1
expression, with a lower rate of grade 3 and 4 adverse
events. The combination of dabrafenib and trametinib
would be an option for patients whose tumor harbors a
BRAF V600 activating mutation, which is not the case
in this patient.

References

Faries MB, Thompson JF, Cochran AJ, et al. Completion


Dissection or Observation for Sentinel-Node Metastasis in
Melanoma. New England Journal of Medicine.
2017;376(23):2211-2222.
https://www.nejm.org/doi/full/10.1056/nejmoa1613210
Weber J, Mandala M, Del Vecchio M, et al. Adjuvant
Nivolumab versus Ipilimumab in Resected Stage III or IV
Melanoma. New England Journal of Medicine.
2017;377(19):1824-1835.
https://www.nejm.org/doi/full/10.1056/NEJMoa170903
0

Incorrect

Question 9:
A 52-year-old man with no previous medical history
presents with a new pigmented lesion on his left arm.
Biopsy shows melanoma, 1.1 mm in thickness, with
ulceration, 1 mitosis per mm^2. BRAF testing is performed
and shows a BRAF V600E mutation. Wide local excision
shows no residual melanoma. Sentinel axillary lymph node
biopsy is negative for melanoma. His ECOG performance
status is 0. Hematologic and chemistry profiles are within
normal limits.
Which of the following is the next best step in
management?

Correct Answer
Observation

Rationale

This is a patient with stage IIA melanoma (T2b N0). No


adjuvant therapy is indicated at this time, except in the
context of a clinical trial.

References

Kaufman HL, Kirkwood JM, Hodi FS, et al. The Society


for Immunotherapy of Cancer consensus statement on
tumour immunotherapy for the treatment of cutaneous
melanoma. Nat Rev Clin Oncol. 2013;10(10):588-598.
https://pubmed.ncbi.nlm.nih.gov/23982524/

Correct

Question 10:
A 65-year-old man presents in the clinic with unintentional
weight loss, palpable right axillary lymph nodes, and
fatigue. The patient has a history of stage IIA malignant
melanoma of the back. Three years ago, a wide local
excision and sentinel lymph node biopsy was completed.
ECOG performance status is 1. Cross-sectional imaging of
the chest, abdomen, and pelvis demonstrates numerous
nodules in the bilateral lung and liver and right axillary
adenopathy suspicious for metastatic disease. Biopsy of a
right axillary lymph node reveals metastatic melanoma.
Tests show BRAF wildtype. Laboratory studies are
unremarkable except for lactate dehydrogenase (LDH) of
500 U/L.
Based on this patient‘s diagnosis and tumor
characteristics, what is the most appropriate option for
first-line treatment?

Correct Answer
Nivolumab with ipilimumab

Rationale

In patients with a good performance status without a


BRAF V600 activating mutation, the best initial choice
for treatment is a combination (PD-1 inhibitor/CTLA-4
inhibitor) or single-agent PD-1 inhibitor therapy (Choice
A). In this case, the patient‘s tumor does not have a
BRAF V600E activating mutation, which precludes
choice C. The combination of PD-1 inhibitor therapy
and CTLA-4 inhibitor therapy showed overall survival
(OS) superiority compared with either agent alone in
CheckMate 067 (Choice B). High-dose Interleukin 2
was the first treatment for metastatic melanoma with a
median OS of 11.4 months. However, given the
improved responses with new agents and higher
toxicity profile of high dose IL2, it is now a less
preferred approach (choice D).

References

Hodi FS, Chiarion-Sileni V, Gonzalez R, et al.


Nivolumab plus ipilimumab or nivolumab alone
versus ipilimumab alone in advanced melanoma
(CheckMate 067): 4-year outcomes of a multicentre,
randomized, phase 3 trial [published correction
appears in Lancet Oncol. 2018 Dec;19(12):e668]
[published correction appears in Lancet Oncol. 2018
Nov;19(11):e581]. Lancet Oncol. 2018;19(11):1480-
1492. doi:10.1016/S1470-2045(18)30700-9

Weber JS, Gibney G, Sullivan RJ, et al. Sequential


administration of nivolumab and ipilimumab with a
planned switch in patients with advanced melanoma
(CheckMate 064): an open-label, randomized, phase 2
trial [published correction appears in Lancet Oncol.
2016 Jul;17 (7):e270]. Lancet Oncol. 2016;17(7):943-
955. doi:10.1016/S1470-2045(16)30126-7

Correct

Question 11:
A 51-year-old woman with a history of melanoma of the
abdomen approximately 15 years ago treated with wide
local excision presents a palpable nodule on her right
flank. Excision is performed, and the pathology is
consistent with a 2-cm deposit of metastatic melanoma in
the subcutaneous tissue; margins are negative. A staging
PET/CT is negative for other sites of disease. Molecular
testing is negative for the presence of a BRAF V600
mutation.
Which of the following is the most appropriate next step?

Your Answer
Close monitoring with clinical and radiographic
imaging
Correct Answer
Adjuvant therapy for one year with pembrolizumab

Rationale

Adjuvant therapy with nivolumab has demonstrated an


improvement in recurrence-free survival (RFS)
compared to ipilimumab; additionally, PD-1 based
therapy has an improved tolerability profile.
Pembrolizumab has also demonstrated an improved
RFS benefit compared to placebo. Targeted therapy
with adjuvant dabrafenib and trametinib is an option for
patients whose tumors harbor a BRAFV600 mutation,
but this patient‘s tumor is negative for BRAFV600.

References

Ascierto PA, Del Vecchio M, Mandalà M, et al.


Adjuvant nivolumab versus ipilimumab in resected
stage IIIB-C and stage IV melanoma (CheckMate 238):
4-year results from a multicentre, double-blind,
randomised, controlled, phase 3 trial. Lancet Oncol.
2020;21(11):1465-1477. doi:10.1016/S1470-
2045(20)30494-0

Eggermont AMM, Blank CU, Mandala M, et al. Longer


Follow-Up Confirms Recurrence-Free Survival Benefit
of Adjuvant Pembrolizumab in High-Risk Stage III
Melanoma: Updated Results From the EORTC 1325-
MG/KEYNOTE-054 Trial. J Clin Oncol.
2020;38(33):3925-3936. doi:10.1200/JCO.20.02110

Incorrect

Question 12:
A 27-year-old woman presented to a dermatologist for a
routine skin examination. The dermatologist noticed a
lesion with scalloped borders on her right arm.
Which additional features would be alarming?

Your Answer
There is no bleeding, itching, or pain.

Correct Answer
The lesion has changed over time.

Rationale

Certain features in skin lesions are associated with an


increased risk of a malignant melanoma diagnosis.
Change in a pre-existing lesion is very alarming.
Therefore, even in a lesion that may look concerning,
stability and lack of change in its appearance over time
are less suggestive for developing melanoma.
Bleeding, itching, or pain associated with a lesion are
also very concerning for both a diagnosis of melanoma
and more advanced melanoma; therefore, the absence
of these is less concerning. Melanomas can lack
pigment, so all changing lesions, regardless of color,
should be evaluated and biopsied, and lack of pigment
itself is not a concerning feature. Surrounding lesions
can be common in patients, and a lesion that does not
look like the others (the ugly duckling) or stands out is
concerning.

References

Fears TR, Guerry Dt, Pfeiffer RM, et al. Identifying


individuals at high risk of melanoma: a practical
predictor of absolute risk. J Clin Oncol.
2006;24(22):3590-3596
Incorrect

Question 13:
A 75-year-old man presented to a dermatologist after
noting multiple pigmented nodules on his scalp. Three
years prior, he was diagnosed with a 2.60-mm Breslow,
Clark level III, focally ulcerated, and superficial spreading
melanoma. Additionally, testing found 1 mitosis/mm^2, a
partial regression, moderate tumor-infiltrating lymphocytes
(TIL) with no evidence of satellitosis, angiolymphatic, or
perineural invasion. A sentinel lymph node biopsy was
negative at that time. Today a shave biopsy is performed
on one of the nodules, and the pathology notes malignant
melanoma, with no overlying connection to the epidermis,
consistent with a dermal metastasis.
Which of the following mutations is most likely to be found
on genetic analysis of the primary specimen?

Your Answer
GNA11 c.547C>T (R183C) mutation

Correct Answer
NRAS c.38G>T (G13V) mutation

Rationale

BRAF exon 15 (37 to 50%), NRAS codon 61 (13 to


25%), and NF1 (11.9%) mutations account for the
majority of mutations found in cutaneous melanoma.
Based on its association with chronically sun-damaged
(CSD) skin, cutaneous melanoma can be categorized
as CSD or non-CSD melanoma. CSD and non-CSD
melanoma have different clinicopathological
characteristics and are associated with different driver
mutations. CSD melanomas typically arise in older
(>55 years) patients on sun-exposed areas
(head/neck, dorsal surfaces of distal extremities). They
are associated with specific driver mutations
(BRAFnonV600E, NRAS, NF1, or KIT) and genetic
signatures of UV-induced DNA damage [G > T (UVA)
or C > T (UVB)] transitions. In contrast, non-CSD
melanomas typically arise in younger (<55 years)
patients on intermittently sun-exposed areas (trunk,
proximal extremities), and are associated with
BRAFV600E/K driver mutations and often lack genetic
signature of UV mutagenesis. Mutations in GNA11 and
GNAQ are commonly observed in uveal melanoma but
not in cutaneous melanoma. KRAS codon 12
mutations are commonly (15 to 25%) observed in lung
cancer but not in melanoma. PIK3CA exon nine
mutations are observed in up to 26% of breast cancer.
Although PTEN loss is seen in 7 to 28% of melanoma,
these mutations typically occur in exon 5 rather than
exon 9.

References

Curtin JA, Fridlyand J, Kageshita T, et al. Distinct


Sets of Genetic Alterations in Melanoma. New
England Journal of Medicine. 2005;353(20):2135-2147

Hodis E, Watson IR, Kryukov GV, et al. A landscape of


driver mutations in melanoma. Cell. 2012;150(2):251-
263

Incorrect

Question 14:
A 64-year-old woman presented with a pigmented and
ulcerated lesion on her left leg and palpable left inguinal
lymph nodes. She underwent wide local excision of the
lesion with complete lymph node dissection of the left
groin. Final pathology showed a 3-mm thick ulcerated
lesion, and two of 10 lymph nodes were positive for
melanoma with no extracapsular extension. The largest
tumor deposit in the lymph nodes was 7 mm. BRAF
mutation was negative. She is now healed and ready for
additional treatment if needed.
Which of the following should you recommend now?

Your Answer
Immunotherapy, followed by radiation therapy to the
groin

Correct Answer
Immunotherapy

Rationale

The only appropriate treatment for this patient is


immunotherapy. Radiation therapy is not indicated
after complete lymph node dissection unless there are
high-risk features for locoregional recurrence (multiple
lymph nodes involved, matted lymph nodes, and
extracapsular extension). Radiation to the groin or the
primary site is not considered appropriate.

References

Henderson MA, Burmeister BH, Ainslie J, et al.


Adjuvant lymph-node field radiotherapy versus
observation only in patients with melanoma at high
risk of further lymph-node field relapse after
lymphadenectomy (ANZMTG 01.02/TROG 02.01): 6-
year follow-up of a phase 3, randomised controlled
trial. Lancet Oncol. 2015;16(9):1049-1060.
doi:10.1016/S1470-2045(15)00187-4

Daniels CP, Liu HY, Porceddu SV. Indications and


limits of postoperative radiotherapy for skin
malignancies. Curr Opin Otolaryngol Head Neck Surg.
2021;29(2):100-106.
doi:10.1097/MOO.0000000000000696

Incorrect

Question 15:
A 56-year-old woman was seen after wide local excision
and sentinel lymph node biopsy of a dark ulcerated lesion
on her back. Final pathology showed malignant melanoma
4.5 mm in depth with negative margins. The sentinel
lymph node biopsy from her left axillary area was negative.
BRAF mutation was positive. Her disease stage grouping
was stage IIC. Clinically she has healed well and is ready
to resume work. She would like to know if there is anything
else that she can do to improve her chances of preventing
cancer recurrence.
Which of the following should you recommend now?

Your Answer
High-dose interferon

Correct Answer
Pembrolizumab

Rationale

Up until recently, the only approved adjuvant treatment


for this stage was high-dose interferon. Recently, the
completion of KEYNOTE-716 changed the landscape.
This randomized controlled trial demonstrated a
statistically significant improvement in recurrence-free
survival (RFS) at the time of the first interim analysis
for patients randomly selected to the pembrolizumab
arm compared with placebo, with a hazard ratio of 0.65
(95% CI 0.46-0.92; P = 0.0132). The median RFS was
not reached in either arm. This study led to the
approval of pembrolizumab in this indication. High-
dose interferon used to be the standard of care but has
rarely been used due to its toxicity. Dabrafenib and
trametinib have not been studied in this indication.
Observation is still an option for patients who decline to
receive pembrolizumab. This patient would like to
decrease her risk of disease recurrence, hence
observation is not appropriate for her.

References

Luke JJ, Rutkowski P, Queirolo P, et al. LBA3


Pembrolizumab versus placebo after complete
resection of high-risk stage II melanoma: Efficacy and
safety results from the KEYNOTE-716 double-blind
phase III trial. Ann Oncol. 2021;32 (suppl_5): S1314-
S1315. doi: 10.1016/j.annonc.2021.08.2116

Poklepovic AS, Luke JJ. Considering adjuvant


therapy for stage II melanoma. Cancer.
2020;126(6):1166-1174. doi: 10.1002/cncr.32585

Incorrect

Question 16:
A 45-year-old woman presents with generalized seizures
and was found to have a solitary right frontal lobe mass
with perilesional edema and midline shift. She had
reported progressively worsening headaches over the past
six weeks. Her medical history consists of a stage II
BRAF-mutated melanoma that was resected from her
back a year ago. At that time, she had a wide local
excision and sentinel lymph node biopsy of the right axilla.
Antiseizure therapy and dexamethasone were started.
Which of the following is the most appropriate next step?

Your Answer
Dabrafenib and trametinib

Correct Answer
Surgical resection

Rationale

Upfront surgical resection or radiation therapy for


symptomatic brain metastases is the standard of care.
Immunotherapy or targeted therapy can be used
upfront only in small (3 cm) asymptomatic brain
metastases with close monitoring for early detection of
progressing brain metastasis, which would require
surgery or radiation therapy. Temozolomide has no
role in treating melanoma brain metastasis.

References

Long GV, Atkinson V, Lo S, et al. Combination


nivolumab and ipilimumab or nivolumab alone in
melanoma brain metastases: a multicentre
randomised phase 2 study. Lancet Oncol.
2018;19(5):672-681. doi:10.1016/S1470-2045(18)30139-
6

Tawbi HA, Forsyth PA, Hodi FS, et al. Long-term


outcomes of patients with active melanoma brain
metastases treated with combination nivolumab plus
ipilimumab (CheckMate 204): final results of an open-
label, multicentre, phase 2 study. Lancet Oncol.
2021;22(12):1692-1704. doi:10.1016/S1470-
2045(21)00545-3

Incorrect

Question 17:
A 57-year-old woman is referred to medical oncology for
evaluation of metastatic cancer. She has a history of a
right ocular (uveal) melanoma that was treated with
removal of the right eye (enucleation) five years ago.
Surveillance imaging showed multiple small hypodense
lesions in both liver lobes, concerning for metastasis. She
is asymptomatic. Physical examination shows no
hepatomegaly or abdominal tenderness. Laboratory test
results including complete blood count and liver function
tests are within normal limits. Liver biopsy confirms
metastatic melanoma. Mutational profiling of the tumor
shows mutations in GNAQ and BAP1 genes, indicating
melanoma of uveal origin. Additional testing reveals the
patient to be positive for the HLA-A*02:01 allele.
Which of the following treatment options is most
appropriate for this patient?

Correct Answer
Tebentafusp

Rationale

Uveal melanoma has a distinct disease biology, as


compared to cutaneous melanoma. Mutations in the
GNAQ or GNA11 genes are most common in primary
uveal melanoma, with subsequent mutations in the
BAP1 or SF3B1 genes associated with metastasis.
Uveal melanoma does not have mutations in the BRAF
V600 genes that can be therapeutically targeted in
cutaneous melanoma. Metastatic uveal melanoma is
also poorly responsive to immune checkpoint inhibitor
therapy. Tebentafusp (IMCgp100) is a novel, bispecific
protein that binds the CD3 receptor on T cells and
engages gp100 (a protein expressed by normal
melanocytes and melanoma cells) on tumor cells. The
expression of the HLA allele, HLA-A*02:01, is
necessary for presentation of gp100 on the cell
surface. In a randomized phase 3 study, patients with
metastatic uveal melanoma were randomly selected to
receive tebentafusp versus investigators‘ choice of
therapy (pembrolizumab, ipilimumab, or dacarbazine).
Patients receiving tebentafusp demonstrated improved
one-year overall survival (73% versus 59%) and six-
month progression-free survival (31% versus 19%),
leading to FDA approval of tebentafusp for metastatic
uveal melanoma patients who are HLA-A*02:01-
positive. In the phase 3 trial by Nathan et al.,
tebentafusp was superior to pembrolizumab or
ipilimumab in patients with metastatic uveal melanoma.
Cytotoxic chemotherapy has very limited efficacy in
metastatic uveal melanoma. In the phase 3 SUMIT
trial, use of dacarbazine had a 0% objective response
rate.

References

Nathan P, Hassel JC, Rutkowski P, et al. Overall


Survival Benefit with Tebentafusp in Metastatic Uveal
Melanoma. N Engl J Med. 2021;385(13):1196-1206.
doi:10.1056/NEJMoa2103485

Algazi AP, Tsai KK, Shoushtari AN, et al. Clinical


outcomes in metastatic uveal melanoma treated with
PD-1 and PD-L1 antibodies. Cancer.
2016;122(21):3344-3353. doi:10.1002/cncr.30258

Carvajal RD, Piperno-Neumann S, Kapiteijn E, et al.


Selumetinib in Combination With Dacarbazine in
Patients With Metastatic Uveal Melanoma: A Phase III,
Multicenter, Randomized Trial (SUMIT) [published
correction appears in J Clin Oncol. 2018 Dec
10;36(35):3528]. J Clin Oncol. 2018;36(12):1232-1239.
doi:10.1200/JCO.2017.74.1090

Correct

Question 18:
A 74-year-old man was referred to see you after being
diagnosed with a neuroendocrine cancer on his scalp. He
had a flesh-colored lesion on his scalp that grew over the
last three months; excisional biopsy showed small, round,
blue cells with sparse cytoplasm and abundant mitoses.
Differential diagnosis includes Merkel cell carcinoma or
metastatic visceral neuroendocrine carcinoma to the skin.
Which of the following immunohistochemistry results is
most consistent with Merkel cell carcinoma?

Your Answer
CK20-negative, TTF1 negative

Correct Answer
CK20-positive, TTF1 negative

Rationale

It is difficult to differentiate between primary Merkel cell


carcinoma and metastatic neuroendocrine carcinoma
(eg, metastatic small cell lung cancer).
Immunohistochemistry is helpful in differentiating
between the two and informing a diagnosis of Merkel
cell carcinoma. CK20 is positive in most cases of
Merkel cell carcinoma, whereas TTF1 is almost always
negative in Merkel cell carcinoma. TTF1 is positive in
approximately 80% of small cell lung cancer cases.
Metastatic small cell lung cancer typically
demonstrates CK20-negative and TTF1-positive
histology in 80% of cases.

References

Leech SN, Kolar AJ, Barrett PD, Sinclair SA, Leonard


N. Merkel cell carcinoma can be distinguished from
metastatic small cell carcinoma using antibodies to
cytokeratin 20 and thyroid transcription factor 1. J
Clin Pathol. 2001;54(9):727-729.
doi:10.1136/jcp.54.9.727

Chan JK, Suster S, Wenig BM, Tsang WY, Chan JB,


Lau AL. Cytokeratin 20 immunoreactivity
distinguishes Merkel cell (primary cutaneous
neuroendocrine) carcinomas and salivary gland small
cell carcinomas from small cell carcinomas of various
sites. Am J Surg Pathol. 1997;21(2):226-234.
doi:10.1097/00000478-199702000-00014

Incorrect

Question 19:
A 56-year-old woman presented to the oncology clinic with
a recently diagnosed a 1-mm deep, ulcerated melanoma
in the right thigh. She underwent primary resection and
sentinel lymph node resection revealing one (of one)
positive lymph nodes with 1 mm of tumor involvement in
the node. PET-CT imaging showed no evidence of distant
disease. She was followed with serial nodal
ultrasonography every three months. After one year,
ultrasonogram demonstrated multiple abnormal lymph
nodes in the right side of her groin. Biopsy of a lymph
node was consistent with metastatic melanoma
histologically similar to the original melanoma.
Which of the following should you do now?

Your Answer
Pembrolizumab

Correct Answer
Nodal ultrasonogram every three months

Rationale

According to the MSLT II trial, full nodal dissection can


be avoided in patients who are diagnosed with
localized melanoma that is clinically negative nodal
with positive sentinel lymph nodes, provided they
receive adequate follow-up consisting of observation
and nodal ultrasonography every three months. The
omission of full nodal dissection in patients with clinical
surveillance showed no negative impact on melanoma-
specific survival (dissection arm: 86% +/- 1.3%;
observation arm: 86% +/- 1.2%) and lowered the risk
for lymphedema from 21.4% to 6%. In the MSLT II trial,
complete lymph node dissection was reserved for
recurrence of nodal melanoma while on active
surveillance. Pembrolizumab is approved for adjuvant
treatment in patients who have lymph node
involvement after complete resection.

References

Faries MB, Thompson JF, Cochran AJ, et al.


Completion Dissection or Observation for Sentinel-
Node Metastasis in Melanoma. N Engl J Med.
2017;376(23):2211-2222. doi:10.1056/NEJMoa1613210

Leiter U, Stadler R, Mauch C, et al. Final Analysis of


DeCOG-SLT Trial: No Survival Benefit for Complete
Lymph Node Dissection in Patients With Melanoma
With Positive Sentinel Node. J Clin Oncol.
2019;37(32):3000-3008. doi:10.1200/JCO.18.02306

Incorrect

Question 20:
A 48-year-old man presents to the dermatology clinic with
a mole on his right forearm that has been painless but
growing. He is otherwise feeling well and healthy. No other
concerning skin lesions are noted and physical
examination findings are otherwise unremarkable. Biopsy
of the mole shows melanoma, 5.1 mm in depth, without
ulceration. The patient undergoes a sentinel lymph node
biopsy, with two of three axillary lymph nodes showing
involvement by melanoma. Molecular testing shows
wildtype BRAF.
Which of the following is the most appropriate next step?

Your Answer
Observation with serial nodal ultrasonogram

Correct Answer
Serial nodal ultrasonogram and adjuvant anti-PD1
therapy

Rationale

Adjuvant immunotherapy has clearly demonstrated


significant recurrence-free survival benefit versus
observation alone for resected stage III melanoma.
This is considered the current standard of care.
Multiple studies (MSLT-1, MSLT-2, DeCOG-SLT) have
examined the role of completion lymph node dissection
(CLND) in patients with positive sentinel lymph nodes
(but were clinically N0), and none of these studies
have shown benefit in melanoma-specific or overall
survival. CLND does offer local control, but this must
be balanced against the significant morbidity of
surgery, and thus should not be considered the default
option for patients.

References

Eggermont AMM, Blank CU, Mandala M, et al.


Adjuvant pembrolizumab versus placebo in resected
stage III melanoma. N Engl J Med. 2018;378(19):1789-
1801. doi:10.1056/NEJMoa1802357

Faries MB, Thompson JF, Cochran AJ, et al.


Completion dissection or observation for sentinel-
node metastasis in melanoma. N Engl J Med.
2017;376(23):2211-2222. doi:10.1056/NEJMoa1613210

Incorrect

Question 21:
A 53-year-old man was diagnosed with metastatic
melanoma to the lungs. His melanoma had a BRAF
V600E mutation. He underwent treatment with ipilimumab
and nivolumab therapy, with subsequent disease
progression to his liver. His treatment was then switched
to BRAF and MEK inhibitor therapy with dabrafenib and
trametinib. Two months into this treatment, his repeat
scans showed decrease in all of his sites of metastases.
One week later, he presented with decreased visual acuity
in his left eye. He was seen by an ophthalmologist, with
optical coherence tomography (OCT) demonstrating a
serous retinal detachment. He was advised to hold
treatment, and his visual symptoms resolved entirely in
one week, with repeat OCT showing improvement of the
detachment.
Which of the following should you do now?

Your Answer
Permanently discontinue dabrafenib and trametinib

Correct Answer
Resume dabrafenib and trametinib, with
consideration for a dose reduction

Rationale

Retinal detachments are a known adverse effect of


MEK inhibitors, and generally do not cause irreversible
loss of vision or serious eye damage. If retinal
detachment improves within three weeks after therapy
interruption, trametinib may be resumed at either the
same or lower dose. If it does not improve within three
weeks after therapy interruption, trametinib dose may
be reduced or permanently discontinued. There would
be no indication to discontinue dabrafenib, as BRAF
inhibitors are unlikely to cause this symptom. Adding
nivolumab to dabrafenib alone would also not be
indicated given the possibility of continuing trametinib,
patient‘s treatment response to the current regimen,
and prior disease progression on nivolumab.

References

Francis JH, Habib LA, Abramson DH, et al. Clinical


and morphologic characteristics of MEK inhibitor-
associated retinopathy: differences from central
serous chorioretinopathy. Ophthalmology.
2017;124(12):1788-1798.
doi:10.1016/j.ophtha.2017.05.038
Méndez-Martínez S, Calvo P, Ruiz-Moreno O, et al.
Ocular adverse events associated with MEK
inhibitors. Retina. 2019;39(8):1435-1450.
doi:10.1097/IAE.0000000000002451

Incorrect

Question 22:
A 45-year-old man presented with a growing lesion on his
right upper arm. Biopsy of the lesion revealed melanoma
with a Breslow depth of 4.2 mm and ulceration. PET-CT
scan and brain MRI did not show any distant metastases.
He underwent a wide local excision and
lymphoscintigraphy, with pathology showing complete
excision of melanoma. Two sentinel lymph nodes from the
right axilla were removed, both with 1-mm deposit of
melanoma. The tumor did not show a BRAF V600
mutation. His Eastern Cooperative Oncology Group
performance status is 0.
Which of the following should you recommend now?

Correct Answer
Adjuvant anti-PD1 immunotherapy

Rationale

This patient has stage IIIC (T4bN2a) melanoma.


Currently, adjuvant nivolumab and pembrolizumab are
FDA-approved treatments for these patients, along
with adjuvant dabrafenib and trametinib, for patients
with BRAF-mutant disease. This patient‘s tumor does
not have a BRAF mutation. Given the high risk of
relapse with stage IIIC melanoma postsurgery,
adjuvant anti-PD1 therapy would be recommended,
and has shown superior relapse-free survival
compared to placebo/observation for these patients.
Results from the MSLT-2 and DeCOG-SLT studies
demonstrate that completion lymph node dissection
(CLND) is no longer considered the standard
recommended treatment for all patients with positive
sentinel lymph node biopsy specimens. Adjuvant
radiation would have no role in sentinel lymph node-
positive melanoma.

References

Eggermont AMM, Blank CU, Mandala M, et al.


Adjuvant pembrolizumab versus placebo in resected
stage iii melanoma. N Engl J Med. 2018;378(19):1789-
1801. doi:10.1056/NEJMoa1802357

Ascierto PA, Del Vecchio M, Mandalà M, et al.


Adjuvant nivolumab versus ipilimumab in resected
stage IIIB-C and stage IV melanoma (CheckMate 238):
4-year results from a multicentre, double-blind,
randomised, controlled, phase 3 trial [published
correction appears in Lancet Oncol. 2021
Oct;22(10):e428]. Lancet Oncol. 2020;21(11):1465-
1477. doi:10.1016/S1470-2045(20)30494-0

Correct

Question 23:
A 51-year-old man presented to dermatology after noticing
a dark mole on his right arm growing gradually for the past
few months. An excisional biopsy showed at least a 2.5-
mm melanoma with a narrow margin (0.5 mm) of
unaffected tissue. The patient has no other suspicious skin
lesions and no palpable adenopathy on the physical
examination. He feels well with no other symptoms.
Laboratory test results are unremarkable.
Which of the following is the most appropriate next step?

Your Answer
Refer for Mohs surgery

Correct Answer
Wide local excision and sentinel lymph node biopsy

Rationale

Wide local excision is recommended in melanoma; for


patients with disease more than 2 mm in depth (T3a
and above), clinical margins of more than 2 cm are
recommended. Biopsy-based staging of intermediate-
thickness or thick primary melanomas provides
important prognostic information and identifies patients
with nodal metastases. Mohs surgery is not
recommended for resecting melanoma of this
thickness. Patients with intermediate-thickness
melanoma without gross lymph node involvement
would first require a sentinel lymph node biopsy and a
physical examination or ultrasonogram for prognostic
purposes.

References

National Comprehensive Cancer Network. NCCN


Clinical Practice Guidelines in Oncology: Melanoma:
Cutaneous. Version 2. 2022.
https://www.nccn.org/professionals/physician_gls/pdf
/cutaneous_melanoma.pdf

Morton DL, Thompson JF, Cochran AJ, et al. Final


trial report of sentinel-node biopsy versus nodal
observation in melanoma. N Engl J Med.
2014;370(7):599-609. doi:10.1056/NEJMoa1310460
Incorrect

Question 24:
A 54-year-old woman presented to the emergency
department with a headache. MRI of the brain showed two
brain lesions concerning for metastases but no edema.
She had history of stage II melanoma five years prior,
treated with wide local excision and sentinel lymph node
biopsy. Headache improved with pain medications. PET-
CT showed hypermetabolic bilateral lung nodules.
Pathology findings from CT-guided needle biopsy
demonstrated melanoma with wildtype BRAF and NRAS
mutation. Radiation oncology performed stereotactic
radiosurgery (SRS) on the two brain metastases. The
patient is feeling well, with an Eastern Cooperative
Oncology Group performance status of 1. Her headache
has resolved and she has no neurologic deficits.
Laboratory findings showed an elevated lactate
dehydrogenase (LDH) level, but are otherwise
unremarkable.
Which of the following is the most appropriate next step?

Correct Answer
Ipilimumab and nivolumab

Rationale

The patient has stage IV melanoma. Brain metastases


were symptomatic and controlled on pain meds and
SRS. Next steps would include systemic therapy with
ipilimumab and nivolumab, which has activity against
extracranial as well as intracranial disease. After local
treatment of one to three melanoma brain metastases,
adjuvant whole-brain radiation does not provide clinical
benefit in terms of distant intracranial control, survival,
or preservation of performance status. The patient
does not have a BRAF V600 mutation, therefore,
dabrafenib and trametinib are not recommended.
Trametinib monotherapy is rarely used in the standard
management of metastatic melanoma; it has very
modest activity in patients with NRAS mutations and is
not recommended in first-line treatment of metastatic
melanoma.

References

Hong AM, Fogarty GB, Dolven-Jacobsen K, et al.


Adjuvant whole-brain radiation therapy compared
with observation after local treatment of melanoma
brain metastases: a multicenter, randomized phase III
trial. J Clin Oncol. 2019;37(33):3132-3141.
doi:10.1200/JCO.19.01414

Wolchok JD, Chiarion-Sileni V, Gonzalez R, et al.


Overall survival with combined nivolumab and
ipilimumab in advanced melanoma [published
correction appears in N Engl J Med. 2018 Nov
29;379(22):2185]. N Engl J Med. 2017;377(14):1345-
1356. doi:10.1056/NEJMoa1709684

Tawbi HA, Forsyth PA, Algazi A, et al. Combined


Nivolumab and ipilimumab in melanoma metastatic to
the brain. N Engl J Med. 2018;379(8):722-730.
doi:10.1056/NEJMoa1805453

Correct

Question 25:
A 42-year-old woman presents to your clinic after being
recently diagnosed with metastatic melanoma. PET-CT
showed lung and liver metastases. She has no relevant
medical history. Other than some fatigue and shortness of
breath on exertion, she is asymptomatic. Her Eastern
Cooperative Oncology Group performance status is 1.
Laboratory testing reveals a mildly elevated lactate
dehydrogenase (LDH) level of 300 U/L; otherwise,
laboratory findings are unremarkable. MRI of the brain
shows no intracranial metastases. Genetic testing
demonstrates a BRAF V600E mutation.
Which of the following is the most appropriate next step?

Your Answer
Dabrafenib

Correct Answer
Ipilimumab and nivolumab

Rationale

The phase 3 trial CheckMate-067 showed significantly


longer overall survival occurred in patients receiving
combination therapy with nivolumab plus ipilimumab
than with ipilimumab alone. Another phase 3 clinical
trial (ECOG 6134) compared ipilimumab and
nivolumab to dabrafenib and trametinib in first-line
treatment for BRAF V600–mutated advanced
melanoma and showed that ipilimumab and nivolumab
was associated with improved overall survival. The
DREAMseq phase 3 trial (ECOG 6134) showed that
dabrafenib and trametinib had inferior overall survival
when compared with ipilimumab and nivolumab in first-
line treatment for advanced BRAF V600–mutated
melanoma. Monotherapy with either dabrafenib or
ipilimumab is inferior to combined regimens dabrafenib
and trametinib, and ipilimumab and nivolumab,
respectively.
References

Larkin J, Chiarion-Sileni V, Gonzalez R, et al. Five-


year survival with combined nivolumab and
ipilimumab in advanced melanoma. N Engl J Med.
2019;381(16):1535-1546. doi:10.1056/NEJMoa1910836

Atkins MB, Lee SJ, Chmielowski B, et al. DREAMseq


(Doublet, Randomized Evaluation in Advanced
Melanoma Sequencing): a phase III trial—ECOG-
ACRIN EA6134. J Clin Oncol. 2021;39(suppl
36):356154-356154.
doi:10.1200/JCO.2021.39.36_suppl.356154

Incorrect

Question 26:
A 71-year-old man presented to you after being diagnosed
with metastatic Merkel cell carcinoma. He had a growing
flesh-colored lesion on his left thigh. PET-CT was done
and showed hypermetabolic liver lesions. Pathology
findings from CT-guided liver biopsy showed evidence of
metastatic Merkel cell carcinoma. The patient is feeling
well. Eastern Cooperative Oncology Group performance
status of 1. Laboratory test results are unremarkable.
Which of the following should you recommend now?

Your Answer
Carboplatin and etoposide

Correct Answer
Pembrolizumab
Rationale

Pembrolizumab was studied in a phase 2 trial in


patients with advanced Merkel cell carcinoma.
Pembrolizumab demonstrated durable tumor control, a
generally manageable safety profile, and good overall
survival compared with historical data from patients
treated with first-line chemotherapy. Chemotherapy
with platinum and etoposide was the historical
standard of care for metastatic Merkel cell carcinoma;
it was sensitive to chemotherapy regimens used in
small cell lung cancer, but those responses were
usually brief. Outcomes observed with anti-PD1
therapy are better with durable responses. No data are
available in comparing ipilimumab and nivolumab to
anti-PD-1 monotherapy; this combination of is
associated with a higher rate of immune adverse
events and is therefore not recommend in this setting.

References

Nghiem P, Bhatia S, Lipson EJ, et al. Durable tumor


regression and overall survival in patients with
advanced Merkel cell carcinoma receiving
pembrolizumab as first-line therapy. J Clin Oncol.
2019;37(9):693-702. doi:10.1200/JCO.18.01896

Pectasides D, Moutzourides G, Dimitriadis M,


Varthalitis J, Athanassiou A. Chemotherapy for
Merkel cell carcinoma with carboplatin and etoposide.
Am J Clin Oncol. 1995;18(5):418-420.
doi:10.1097/00000421-199510000-00011

Incorrect
Question 27:
A 56-year-old man presents with an abnormal mole in the
left forearm. Biopsy demonstrates a malignant melanoma
that is 1.5 mm in Breslow depth and not ulcerated. The
patient is otherwise asymptomatic. The next step planned
in staging and treatment is a wide local excision of the
melanoma.
What other intervention is most appropriate at this time for
staging?

Your Answer
PET imaging

Correct Answer
Sentinel lymph node biopsy

Rationale

Sentinel lymph node status is an independent predictor


of overall survival and is an important component of
staging and prognosis. All patients at risk for regional
nodal involvement (at least T1b disease) should be
offered this procedure. PET and cross-sectional
imaging are controversial in early-stage disease and
are not an independent predictor of overall survival.
Wide local excision alone is not recommended in
patients at risk for regional nodal involvement as it
does not provide accurate staging.

References

Swetter SM, Tsao H, Bichakjian CK, et al. Guidelines


of care for the management of primary cutaneous
melanoma. J Am Acad Dermatol. 2019;80(1):208-250.
doi:10.1016/j.jaad.2018.08.055

Wong SL, Faries MB, Kennedy EB, et al. Sentinel


lymph node biopsy and management of regional
lymph nodes in melanoma: American Society of
Clinical Oncology and Society of Surgical Oncology
clinical practice guideline update. J Clin Oncol.
2018;36(4):399-413. doi:10.1200/jco.2017.75.7724

Morton DL, Thompson JF, Cochran AJ, et al. Final


trial report of sentinel-node biopsy versus nodal
observation in melanoma. N Engl J Med.
2014;370(7):599-609. doi:10.1056/nejmoa1310460

Incorrect

Question 28:
A 78-year-old man presents with a pigmented lesion on
the upper right side of his back that has been growing in
size, as well as getting darker, with irregular margins over
the last six months. He also mentions that he has a lump
in his right armpit, which he noticed two months ago, that
has been increasing in size with intermittent bleeding.
Biopsy of the lesion and pathology reports malignant
melanoma. He underwent a staging PET/CT, which
showed a hyperpigmented lesion on his upper back along
with hypermetabolic axillary lymphadenopathy and
multiple liver metastases. His biopsy sample was sent for
next-generation sequencing.
In which of the following genes would you most likely
expect to see a mutation?

Your Answer
BRAF V600K

Correct Answer
BRAF V600E
Rationale

BRAF activating mutations are present in


approximately 40% to 60% of patients with metastatic
cutaneous melanomas, among them BRAF V600E is
more common compared to BRAF V600K, accounting
for 80% to 90% of cases and 15% of cases,
respectively. Other less frequent BRAF mutations
account for less than 5% of cases. NRAS mutations
occur in about 15% to 20% of cases. KIT mutations are
more common in patients with chronic sun damage,
compared to the above mutations which are more
common in patients without chronic sun damage.
Generally, these mutations are mutually exclusive
except in rare cases.

References

Davies H, Bignell GR, Cox C, et al. Mutations of the


BRAF gene in human cancer. Nature.
2002;417(6892):949-954. doi:10.1038/nature00766

Curtin JA, Fridlyand J, Kageshita T, et al. Distinct sets


of genetic alterations in melanoma. N Engl J Med.
2005;353(20):2135-2147. doi:10.1056/NEJMoa050092

Hodis E, Watson IR, Kryukov GV, et al. A landscape of


driver mutations in melanoma. Cell. 2012;150(2):251-
263. doi:10.1016/j.cell.2012.06.024

Lin YC, Chang YM, Ho JY, et al. C-kit expression of


melanocytic neoplasm and association with
clinicopathological parameters and anatomic
locations in Chinese people. Am J Dermatopathol.
2013;35(5):569-575.
doi:10.1097/DAD.0b013e318279566a

Incorrect
Question 29:
A 23-year-old woman was found to have a stage I
melanoma of the left lower back, for which she had a wide
local resection. She has had two prior stage I cutaneous
melanomas resected in the past. Her father has recently
been diagnosed with pancreatic cancer. Given her
personal and family history, you consider the possibility of
a germline mutation and discuss referral to a genetic
counselor for consultation and testing.
A mutation in which of the following genes is most likely to
be identified in this patient?

Correct Answer
CDKN2A

Rationale

A mutation in CDKN2A gene (p16) is the most


commonly identified mutation in familial forms of
melanoma. It also increases the risk of developing
pancreatic cancer. Germline mutations in the BRCA1-
associated protein 1 (BAP1) gene have been identified
in families with cutaneous and uveal melanoma,
though would be less common in the described clinical
context. TP53 germline mutations are associated with
Li-Fraumeni syndrome, in which there is an increased
risk of multiple cancers; increased risk of melanoma
has been reported although not as commonly as with a
CDKN2A mutation. An association between
BRCA1/BRCA2 mutations and melanoma has been
noted in some but not all studies.

References

Begg CB, Orlow I, Hummer AJ, et al. Lifetime risk of


melanoma in CDKN2A mutation carriers in a
population-based sample. J Natl Cancer Inst.
2005;97(20):1507-1515. doi:10.1093/jnci/dji312

Berwick M, Orlow I, Hummer AJ, et al. The prevalence


of CDKN2A germ-line mutations and relative risk for
cutaneous malignant melanoma: an international
population-based study. Cancer Epidemiol
Biomarkers Prev. 2006;15(8):1520-1525.
doi:10.1158/1055-9965.EPI-06-0270

Njauw CN, Kim I, Piris A, et al. Germline BAP1


inactivation is preferentially associated with
metastatic ocular melanoma and cutaneous-ocular
melanoma families. PLoS One. 2012;7(4):e35295.
doi:10.1371/journal.pone.0035295

Correct

Question 30:
A 56-year-old woman presented to the oncology clinic with
a recently diagnosed a 1-mm deep, ulcerated melanoma
in the right thigh. She underwent primary resection and
sentinel lymph node resection revealing one (of one)
positive lymph nodes with 1 mm of tumor involvement in
the node. PET/CT imaging showed no evidence of distant
disease
Which of the following should you do now?

Correct Answer
Surveillance with nodal ultrasonogram every three
months

Rationale
According to the MSLT II trial, patients who have
localized melanoma with clinically negative nodal
disease and show a positive sentinel lymph node can
forego full nodal dissection with adequate follow-up,
consisting of observation and nodal ultrasonogram
every three months. The omission of full nodal
dissection in patients with clinical surveillance showed
no negative impact no melanoma-specific survival
(86% +/- 1.3% and 86% +/- 1.2%, respectively) and
lowered the risk for lymphedema from 21.4% to 6%.
Once-in-a-lifetime nodal ultrasonogram is not a
recommended strategy. To date, nodal dissection has
not consistently proven any impact on overall survival
in patients with melanoma.

References

Faries MB, Thompson JF, Cochran AJ, et al.


Completion Dissection or Observation for Sentinel-
Node Metastasis in Melanoma. N Engl J Med.
2017;376(23):2211-2222. doi:10.1056/NEJMoa1613210

Leiter U, Stadler R, Mauch C, et al. Final Analysis of


DeCOG-SLT Trial: No Survival Benefit for Complete
Lymph Node Dissection in Patients With Melanoma
With Positive Sentinel Node. J Clin Oncol.
2019;37(32):3000-3008. doi:10.1200/JCO.18.02306

Correct

Question 31:
A 52-year-old man presents with a six-month history of
weight loss and abdominal pain. The patient has a history
of stage II oral mucosal melanoma diagnosed three years
ago, treated with wide local excision and sentinel lymph
node biopsy. Cross-sectional imaging reveals multiple liver
lesions. Biopsy of a liver lesion reveals melanoma
histologically similar to his original mucosal melanoma.
Which of the following mutations is most likely to be found
in this patient‘s tumor?

Your Answer
BRAF

Correct Answer
KIT

Rationale

KIT mutations are more common in mucosal


melanoma (15% of cases) compared to cutaneous
melanoma, whereas BRAF and NRAS mutations are
less common. NF1 mutations are more common in
sun-exposed skin. Patients with metastatic mucosal
melanoma have been treated successfully at times
with immunotherapy, but in general response rates are
lower than in patients with cutaneous melanoma. KIT
targeted therapy has also shown some benefit, but
responses tend to be transient.

References

Gutiérrez-Castañeda LD, Nova JA, Tovar-Parra JD.


Frequency of mutations in BRAF, NRAS, and KIT in
different populations and histological subtypes of
melanoma: a systemic review. Melanoma Res.
2020;30(1):62-70. doi:10.1097/CMR.0000000000000628

Tyrrell H, Payne M. Combatting mucosal melanoma:


recent advances and future perspectives. Melanoma
Manag. 2018;5(3):MMT11. Published 2018 Oct 8.
doi:10.2217/mmt-2018-0003
Incorrect

Question 32:
A 46-year-old man presented to the emergency
department with a new-onset fever (102.2 F [39 C]) and
chills, without any other localizing symptoms. One month
prior, he was diagnosed with a BRAF V600E–mutated
metastatic melanoma to the lung, liver, and bone. He
started treatment with dabrafenib plus trametinib. An
extensive infectious workup revealed no abnormalities.
Which of the following is the most likely etiology of this
patient‘s fever?

Your Answer
Neoplastic fever from melanoma

Correct Answer
Pyrexia from BRAF/MEK inhibitors

Rationale

Pyrexia defined as a temperature of 101.3 F (38.5 C)


or greater is the most common adverse event
associated with dabrafenib plus trametinib in both the
adjuvant and metastatic setting, with an incidence of
around 55%. Most of these cases are grade 1 or 2
events and are the most frequent adverse event
leading to treatment discontinuation. Pyrexia typically
occurs early in treatment, with incidence decreasing
over time. Holding treatment with dabrafenib and
trametinib at the onset of pyrexia will often interrupt the
episode and full-dose BRAF/MEK inhibition can be
resumed. The diagnosis of neoplastic fever is
extremely uncommon in melanoma; organic causes
such as infections and medications should be ruled
out. To date, BRAF/MEK inhibitor therapy has not
been associated with a higher risk of PCP infection.

References

Schadendorf D, Robert C, Dummer R, et al. Pyrexia in


patients treated with dabrafenib plus trametinib
across clinical trials in BRAF-mutant cancers. Eur J
Cancer. 2021;153:234-241.
doi:10.1016/j.ejca.2021.05.005

Robert C, Grob JJ, Stroyakovskiy D, et al. Five-Year


Outcomes with Dabrafenib plus Trametinib in
Metastatic Melanoma. N Engl J Med. 2019;381(7):626-
636. doi:10.1056/NEJMoa1904059

Incorrect

Question 33:
A 59-year-old man with a history of hypertension reports
new-onset itching of a pigmented mole that he had
noticed on his left upper back for about five years. The
patient had also seen the mole increase in size and
change in color. He was evaluated by his dermatologist,
who did a biopsy of this lesion for concerns of cutaneous
melanoma. Biopsy results confirmed the lesion to be
malignant melanoma of 2.9 mm depth, Clark Level IV,
nodular type, three mitosis/ mm^2, ulceration absent, brisk
tumor-infiltrating lymphocytes, regression, and
lymphovascular invasion present. There were no
microsatellites, perineural invasion, and in-transit
metastases. The patient underwent complete excision of
the tumor with negative margins, and a
lymphoscintigraphy procedure resulted in three sentinel
lymph nodes. One of the three lymph nodes showed a
focus of melanoma morphologically and by
immunohistochemistry that measured 2.1 mm. Blood test
results, including lactate dehydrogenase (LDH), are
normal, and a whole-body PET scan and brain MRI
showed no evidence of metastases. The tumor is assigned
AJCC Stage IIIA (T2a, N1a, M0). The tumor testing for
BRAF mutation showed wild-type BRAF. The patient is
referred for consultations to discuss risk stratification and
postoperative treatment recommendations to improve
long-term outcomes.
Which of the following is the most appropriate next step?

Correct Answer
Anti PD-1 antibody treatment for one year

Rationale

Melanoma metastases to regional lymph nodes


represent a critical prognostic factor in intermediate
thickness cutaneous melanoma and confer reduced
melanoma-specific survival. The FDA approved High-
dose interferon alfa for one year due to its modest
recurrence-free survival and marginal overall survival
benefit. The newer immune checkpoint inhibitor agents
have shown unequivocal recurrence-free survival and
overall survival benefit in patients with stage III
melanoma when administered for one year. Anti-PD-1
treatment has been determined to be superior and
safer than anti-CTLA-4 treatment and is a preferred
first choice. The patient‘s lymph node disease burden
also makes this the most appropriate treatment option.
In patients with BRAF-mutated tumors, anti-BRAF plus
anti-MEK treatment for one year is FDA approved
since it improves event-free survival and overall
survival. Our patient does not harbor BRAF mutated
melanoma; anti-BRAF plus anti-MEK treatment is not
recommended. Although an approved option, high-
dose interferon alfa is not favored due to its marginal
benefit and significant side effects. ThereforeÂ, the
correct answer is A.
References

Long GV, Hauschild A, Santinami M, et al. Adjuvant


Dabrafenib plus Trametinib in Stage III BRAF-Mutated
Melanoma. N Engl J Med. 2017;377(19):1813-1823.
PMID: 28891408.

Correct

Question 34:
A 70-year-old man presents with slowly progressive
dyspnea, back pain, and mental status change to the
emergency room. He has a history of malignant melanoma
on the right cheek from 3 years ago, which was resected
with negative margins and was determined to be stage I.
Sentinel lymph nodes were not assessed at that time. In
the emergency department, a brain MRI with and without
intravenous gadolinium revealed a 2-cm lesion in the left
parietal lobe and a 1-cm lesion in the right cerebellar
hemisphere. Both were associated with vasogenic edema.
A CT scan with intravenous contrast of the chest,
abdomen, and pelvis showed extensive liver and bone
metastases and one pulmonary mass measuring 2.5 cm in
diameter at the posterior aspect of the right lower lobe.
The patient was given one dose of dexamethasone 10 mg
IV. A CT-guided lung biopsy of the right lower lobe mass
was performed upon admission to the hospital and
confirmed melanoma.
Which diagnostic test should be requested before
selecting first-line systemic therapy?

Your Answer
FGFR3 gene mutation by PCR
Correct Answer
BRAF gene mutation V600 by PCR

Rationale

Metastatic melanoma with a BRAF gene mutation


V600 by PCR indicates treatment with dual BRAF/MEK
inhibition.

References

Ascierto PA, Dummer R, Gogas HJ, et al. Update on


tolerability and overall survival in COLUMBUS:
landmark analysis of a randomised phase 3 trial of
encorafenib plus binimetinib vs vemurafenib or
encorafenib in patients with BRAF V600-mutant
melanoma. Eur J Cancer. 2020;126:33-44. PMID:
31901705.

Incorrect

Question 35:
A 31-year-old woman presents with a 2.1 mm deep,
ulcerated mitoses 2 per high power field (HPF) melanoma
on the left arm. She undergoes a wide local excision with
2Â cm margins and sentinel lymph node biopsy of the left
axilla. She has 1 of 2 positive sentinel nodes. The positive
sentinel lymph node demonstrates a 0.6 mm focus of cells
with no extracapsular extension. Staging PET/CT is
negative for metastatic disease, and the tumor is BRAF
wild-type.
Which of the following is the most appropriate next step?

Your Answer
Completion axillary lymph node dissection with
adjuvant combined PD1/anti-CTLA4 therapy.

Correct Answer
Ultrasound surveillance of the lymph node basin
every 4 months and offer adjuvant PD1 therapy.

Rationale

This is a patient who presents with stage 3C,


T3bN1aM0 melanoma. Two significant decisions need
to be made in this patient regarding the management
of the lymph node basin and the role of adjuvant
therapy. Basin options would include ultrasound
surveillance every four months or completion of lymph
node dissection regarding the lymph node. The risks
and benefits of this approach should be discussed with
the patient. The DeCOG-SLT Trial and MSLT 2
(Completion Dissection or Observation for Sentinel-
Node Metastasis in Melanoma) found no change in
overall survival with dissection versus observation.
However, with observation, patients are more likely to
have lymph node basin recurrence. Numerous studies,
including "Adjuvant Nivolumab versus Ipilimumab in
resected stage III or IV Melanoma," have
demonstrated a significantly longer recurrence-free
survival with adjuvant therapy. The patient is not a
candidate for adjuvant BRAF/MEKi as the tumor was
BRAF wild type.

References

Leiter U, Stadler R, Mauch C, et al. No Survival Benefit


for Complete Lymph Node Dissection in Patients With
Melanoma With Positive Sentinel Node. J Clin Oncol.
2019 Nov 10;37(32):3000-3008. PMID: 31557067.

Weber J, Mandala M, Del Vecchio M, et al. Adjuvant


Nivolumab versus Ipilimumab in Resected Stage III or
IV Melanoma. N Engl J Med. 2017 Nov 9;377(19):1824-
1835. PMID: 28891423.

Faries MB, Thompson JF, Cochran AJ, et al.


Completion Dissection or Observation for Sentinel-
Node Metastasis in Melanoma. N Engl J Med. 2017
Jun 8;376(23):2211-2222. PMID: 28591523.

Incorrect

Question 36:
A 70-year-old woman with no significant past medical
history presents a three month history of a pink, firm skin
nodule on the right lower leg, which has been slowly
progressing. There is a palpable lymph node in the right
inguinal region, firm, fixed, and non-tender measuring
approximately 2 cm in diameter. She has had no B
symptoms. Punch biopsy of the skin lesion reveals CK20
positive, TTF-1 negative, infiltrative, malignant appearing
cells, with brisk tumor-infiltrating lymphocytes. PET/CT
scan revealed a 2 cm right inguinal lymph node with FDG
uptake and maximum SUV of 10, as well as multiple FDG
avid liver and lung lesions ranging between 1 and 3 cm in
diameter.
Which of the following is the most appropriate next step?

Your Answer
Cisplatin

Correct Answer
Pembrolizumab
Rationale

Immunotherapy with pembrolizumab was approved in


2019 by the FDA for the treatment of advanced Merkel
cell skin cancer based on an overall response rate of
56% (complete response of 24% and partial response
of 32%) in clinical trial Keynote-017 which was
nonrandomized, open-label. Avelumab was approved
in 2017 as well. Chemotherapy can lead to high
response rates but has minimal impact on survival due
to the short progression-free survival associated with
chemotherapy. Cemiplimab-rwlc is approved for
advanced squamous cell skin cancer.

References

Nghiem P, Bhatia S, Lipson EJ, et al. Durable Tumor


Regression and Overall Survival in Patients With
Advanced Merkel Cell Carcinoma Receiving
Pembrolizumab as First-Line Therapy. J Clin Oncol.
2019;37(9):693-702. PMID: 30726175.

Incorrect

Question 37:
A 60-year-old man presents to the oncology clinic to start
treatment for metastatic merkel cell carcinoma after his
disease had progressed on chemotherapy. He had
received six cycles of carboplatin and etoposide with the
initial partial response after the first three cycles and
progression after his sixth cycle. He will start 800 mg
avelumab to be infused intravenously over 60 minutes. His
HIV and hepatitis serologies are negative.
Which of the following is the most appropriate next step?

Your Answer
One liter of normal saline

Correct Answer
Acetaminophen and antihistamine

Rationale

One-quarter of patients who receive avelumab exhibit


a severe infusion reaction; hence, the need for
premedication. Avelumab is the only immune
checkpoint inhibitor that requires premedication. Pre-
chemotherapy hydration is not needed for kidney
protection and ondansetron is not needed for nausea
and vomiting prevention. Dexamethasone should not
be used because of its effect on the immune response
that avelumab is trying to elicit.

References

D‘Angelo SP, Bhatia S, Brohl AS, Hamid O, Mehnert


JM, Terheyden P, Shih KC, Brownell I, Lebb C, Lewis
KD, Linette GP, Milella M, Georges S, Shah P, Ellers-
Lenz B, Bajars M, Gzel G, Nghiem PT. Avelumab in
patients with previously treated metastatic Merkel cell
carcinoma: long-term data and biomarker analyses
from the single-arm phase 2 JAVELIN Merkel 200 trial.
J Immunother Cancer. 2020 May;8(1):e000674. doi:
10.1136/jitc-2020-000674. PMID: 32414862; PMCID:
PMC7239697.

Avelumab: HIGHLIGHTS OF PRESCRIBING


INFORMATION

Incorrect

You might also like